You are on page 1of 34

1. CASE DIGEST OF PROVINCE OF NORTH COTABATO VS.

GRP

FACTS:
On August 5, 2008, the Government of the Republic of the Philippines (GRP) and the MILF, through the Chairpersons of
their respective peace negotiating panels, were scheduled to sign a Memorandum of Agreement on the Ancestral Domain
(MOA-AD) Aspect of the GRP-MILF Tripoli Agreement on Peace of 2001 in Kuala Lumpur, Malaysia.

The MOA-AD was preceded by a long process of negotiation and the concluding of several prior agreements between the
two parties beginning in 1996, when the GRP-MILF peace negotiations began. On July 18, 1997, the GRP and MILF Peace
Panels signed the Agreement on General Cessation of Hostilities. The following year, they signed the General Framework
of Agreement of Intent on August 27, 1998.

Early on, however, it was evident that there was not going to be any smooth sailing in the GRP-MILF peace process.
Towards the end of 1999 up to early 2000, the MILF attacked a number of municipalities in Central Mindanao and, in
March 2000, it took control of the town hall of Kauswagan, Lanao del Norte. 3 In response, then President Joseph Estrada
declared and carried out an "all-out-war" against the MILF.

When President Gloria Macapagal-Arroyo assumed office, the military offensive against the MILF was suspended and the
government sought a resumption of the peace talks. The MILF, according to a leading MILF member, initially responded
with deep reservation, but when President Arroyo asked the Government of Malaysia through Prime Minister Mahathir
Mohammad to help convince the MILF to return to the negotiating table, the MILF convened its Central Committee to
seriously discuss the matter and, eventually, decided to meet with the GRP.

Formal peace talks between the parties were held in Tripoli, Libya from June 20-22, 2001, the outcome of which was the
GRP-MILF Tripoli Agreement on Peace (Tripoli Agreement 2001) containing the basic principles and agenda on the
following aspects of the negotiation: Security Aspect, Rehabilitation Aspect, and Ancestral Domain Aspect. With regard
to the Ancestral Domain Aspect, the parties in Tripoli Agreement 2001 simply agreed "that the same be discussed further by
the Parties in their next meeting.

In 2005, several exploratory talks were held between the parties in Kuala Lumpur, eventually leading to the crafting of the
draft MOA-AD in its final form, which, as mentioned, was set to be signed last August 5, 2008.

ISSUE/S:

A. PROCEDURAL ISSUES:

1. Whether the constitutionality and the legality of the MOA is ripe for adjudication;

2. Whether the petitioners have locus standi in assailing the MOA;

3. Whether the petitions have become moot and academic.

B. SUBSTANTIVE ISSUES

4.Whether there is a violation of the people's right to information on matters of public concern (1987
Constitution, Article III, Sec. 7) under a state policy of full disclosure of all its transactions involving public
interest (1987 Constitution, Article II, Sec. 28) including public consultation under Republic Act No. 7160
(LOCAL GOVERNMENT CODE OF 1991)[;]

5. Whether or not the contents of the MOA-AD violate the Constitution and the laws.

HELD:
A. PROCEDURAL ISSUES

Whether the constitutionality and the legality of the MOA is ripe for adjudication.
The petitions are ripe for adjudication. The failure of respondents to consult the local government units or
communities affected constitutes a departure by respondents from their mandate under E.O. No. 3. Moreover,
respondents exceeded their authority by the mere act of guaranteeing amendments to the Constitution. Any alleged
violation of the Constitution by any branch of government is a proper matter for judicial review.

Whether or not the petitioners have locus standi in assailing the MOA.
As the petitions involve constitutional issues which are of paramount public interest or of transcendental importance,
the Court grants the petitioners, petitioners-in-intervention and intervening respondents the requisite locus standi in
keeping with the liberal stance adopted in David v. Macapagal-Arroyo.

Whether the petitions have become moot and academic.


Contrary to the assertion of respondents that the non-signing of the MOA-AD and the eventual dissolution of the GRP
Peace Panel mooted the present petitions, the Court finds that the present petitions provide an exception to the "moot
and academic" principle in view of (a) the grave violation of the Constitution involved; (b) the exceptional character of
the situation and paramount public interest; (c) the need to formulate controlling principles to guide the bench, the bar,
and the public; and (d) the fact that the case is capable of repetition yet evading review.EDcICT

B. SUBSTATIVE ISSUES:

Whether there is a violation of the people's right to information on matters of public concern (1987 Constitution,
Article III, Sec. 7) under a state policy of full disclosure of all its transactions involving public interest (1987
Constitution, Article II, Sec. 28) including public consultation under Republic Act No. 7160.
The people's right to information on matters of public concern under Sec. 7, Article III of the Constitution is in
splendid symmetry with the state policy of full public disclosure of all its transactions involving public interest under
Sec. 28, Article II of the Constitution. The right to information guarantees the right of the people to demand
information, while Section 28 recognizes the duty of officialdom to give information even if nobody demands. The
complete and effective exercise of the right to information necessitates that its complementary provision on public
disclosure derive the same self-executory nature, subject only to reasonable safeguards or limitations as may be
provided by law.HcaATE
The contents of the MOA-AD is a matter of paramount public concern involving public interest in the highest order. In
declaring that the right to information contemplates steps and negotiations leading to the consummation of the contract,
jurisprudence finds no distinction as to the executory nature or commercial character of the agreement.

Whether or not contents of the MOA-AD violate the Constitution and the laws.
The MOA-AD cannot be reconciled with the present Constitution and laws. Not only its specific provisions but the
very concept underlying them, namely, the associative relationship envisioned between the GRP and the BJE, are
unconstitutional, for the concept presupposes that the associated entity is a state and implies that the same is on its way
to independence.
While there is a clause in the MOA-AD stating that the provisions thereof inconsistent with the present legal
framework will not be effective until that framework is amended, the same does not cure its defect. The inclusion of
provisions in the MOA-AD establishing an associative relationship between the BJE and the Central Government is,
itself, a violation of the Memorandum of Instructions From The President dated March 1, 2001, addressed to the
government peace panel. Moreover, as the clause is worded, it virtually guarantees that the necessary amendments to
the Constitution and the laws will eventually be put in place. Neither the GRP Peace Panel nor the President herself is
authorized to make such a guarantee. Upholding such an act would amount to authorizing a usurpation of the
constituent powers vested only in Congress, a Constitutional Convention, or the people themselves through the process
of initiative, for the only way that the Executive can ensure the outcome of the amendment process is through an undue
influence or interference with that process.
2. Government of the Philippines Islands v. Monte de Piedad GR L 9959, December 13, 1916

FACTS

On June 3, 1863 a devastating earthquake occurred in the Philippines. The Spanish Dominions then provided $400,000.00 as aid for
the victims and it was received by the Philippine Treasury. Out of the aid, $80,000.00 was left untouched; it was then invested in the
Monte de Piedad Bank which in turn invested the amount pieces of jewelry.But when the Philippine government later tried to
withdraw the said amount, the bank cannot provide for the amount. The bank argued that the Philippine government is not an
affected party hence has no right to institute a complaint. Bank argues that the government was not the intended beneficiary of the
said amount.

On June 3, 1863, an Earthquake took place in the Philippine Islands, which was then under the Spanish Crown, that devastated lot
of civilians. Therefore on Oct. 6 of that year, a central relief board was appointed, by authority of the King of Spain, to distribute the
money voluntarily contributed by donors. After a thorough investigation and consideration, the relief board allotted $365703.50 to
the various sufferers name in its resolution.
These were later distributed in accordance with the above mentioned allotments, the sum of $30,299.65, leaving a balance of
$365.403.85 for distribution. Upon the petition of the governing body of the Monte de Piedad, dated February 1, 1833, the
Philippine Government, by order dated the first month, directed its treasured to turn over Monte de Piedad the sum of $80,000 of
relief fund in its installment of 20,000 each. These amounts received on the following dates: February 15, March 12, April 14, and
June 2, 1883, and are still in the possession of Monte de Piedad.
The Attorney General in representation of the Philippine Islands, a file of claim for the $80000 together with interest, for the benefit
of those persons or their heirs appearing in the list of names published in the Official Gazette instituted on May, 3, 1912 by the
Government of the Philippine Islands, represented by the Insular Treasurer, and after due trial in the lower court, judgment was
entered in honor of the plaintiff currency, together with legal interest from February 28, 1912, and cost of cause. The Monte de
Piedad then contended that the present Philippine Government cannot file suit on the ground that the obligation of the former was
wiped out when their was a change of sovereignty.

ISSUES

Does the Philippine Government have the personality to institute the action?

RULING

Yes. In this country, the Government as parens patriae has the right to enforce all charities of public nature by virtue of its general
superintending authority over the public interests, where no other person is entrusted with it. This prerogative of parens patriae is
inherent in the supreme power of the State. It is a most beneficial function, and often necessary to be exercised in the interest of
humanity, and for the prevention of injury to those who cannot protect themselves. Furthermore, it would be impracticable for the
beneficiaries to institute an action or actions either individually or collectively. The only course that can be satisfactorily pursued is
for the Government to again assume control of the fund and devote it tot he object for which it was originally destined.

Philosophical Principle:
In the instant case the Philippine Government is not a mere nominal party because it, in bringing and prosecuting this action, is
exercising its sovereign functions or powers. These principles being based "upon the foundation of the great principle of public
policy" are, in the very nature of things applicable to the Philippine Government

Under the Principle of Parens Patriae, the Philippine Government being the guardian of the rights of the people can represent
the legitimate claimants of the beneficiary and therefore has the capacity to file a suit against the appellant. The Philippine
Government is not merely a nominal party thats why it can bring and prosecute this action by exercising its sovereign powers.
The supreme court then held the right of the government to file the case.
3. LEE vs. DIRECTOR of LANDS

G.R. 128195, Oct. 3, 2001

FACTS

Sometime in March 1936, the Dinglasans sold to Lee Liong, a Chinese citizen, a parcel of land at Roxas City.
In 1948, the Dinglasans filed an action for annulment of sale and recovery of land against the heirs of Lee because of
the constitutional prohibition against aliens acquiring ownership of private agricultural land including residential,
commercial, or industrial land.
The trial court and the Court of Appeals both dismissed the case.
On June 27, 1956, the Supreme Court ruled that both the vendor and vendee violated the constitutional prohibition and
that both are equally guilty (Doctrine of Pari Delicto)
Note: In 1935 Constitution, sale of land to non-Philippine citizens are prohibited; Lee Liong buyer was a Chinese citizen

On July 1, 1968, the Dinglasans filed with the Court of First Instance in Capiz an action for recovery of the SAME
parcel of land. This time, they cited the case of Philippine Banking Corporation vs. Lui She, stating that the sale of the
said land to Lee Liong was void for being violative of the Constitution
On September 23, 1968, the heirs of Lee Liong filed with the trial court a motion to dismiss the case on the ground of
res judicata
On October 10, 1968, and November 9, 1968, the trial court denied the motion
The heirs of Lee Liong elevated the case to the Supreme Court by petition for certiorari
On April 22, 1977, the Supreme Court annulled the orders of the trial court and directed it to dismiss the case, holding
that the suit was barred by res judicata
On September 7,1993 Elizabeth Manuel-Lee (a widow of Lee Liong's heir Lee Bing Hoo) and Pacita Yu Lee (a widow
of Lee Liong's other heir Lee Bun Ting) filed a petition with the RTC Roxas City for the reconstitution of the title of
Lot No. 398 because it was burnt together with all the documents in the Register of Deeds during the war
On June 10, 1994, the RTC granted the petition.
On January 25, 1995, the Socilitor General filed a petition with the CA alleging that the RTC didn't have jurisdiction of
the case contending that the petitioners were not the proper parties since the predecessor Lee Liong did not acquire the
land because he was a Chinese citizen
On April 30, 1996, CA approved Solicitor General's petition
On May 24, 1996, Elizabeth Manuel-Lee and Pacita Yu Lee filed with the Court of Appeals a motion for
reconsideration of the decision
On February 18, 1997, the Court of Appeals denied the motion
Hence, petitioners now appealed to the Supreme Court

Issues:

whether or not the Solicitor General's actions were right


whether or not the sale of the land was valid
whether or not reconstitution of the title is valid

Held:

Although ownership of the land cannot revert to the original sellers, because of the doctrine of pari delicto, the
Solicitor General may initiate an action for reversion or escheat of the land to the State
However, this time the land in case is now in the hands of Filipinos (the heirs of the heirs of Lee Liong) and thus the
flaw in the original transaction is considered cured and the title of transferee is rendered valid
a reconstitution of title is the re-issuance of a new certificate of title lost or destroyed in its original form and
condition, therefore, a separate suit is necessary to thresh out the issue of ownership of the land
WHEREFORE, the Court REVERSES and SETS ASIDE the decision of the Court of Appeals; the Court sets aside the
order of reconstitution of title in Reconstitution Case No. R-1928, Regional Trial Court, Roxas City

4. CASE DIGEST: Tecson vs Comelec

Facts of the Case:

This is a consolidation of three cases raising the issue of citizenship of presidential candidate, Ronald Allan Kelly Poe, also known as
Fernando Poe Jr a.k.a. FPJ. MARIA JEANETTE C. TECSON and FELIX B. DESIDERIO, JR., petitioners, vs.The COMMISSION ON
ELECTIONS, RONALD ALLAN KELLY POE (a.k.a. FERNANDO POE, JR.) and VICTORINO X. FORNIER, respondents. ZOILO
ANTONIO VELEZ, petitioner, vs.RONALD ALLAN KELLEY POE, a.k.a. FERNANDO POE, JR., respondent and VICTORINO X.
FORNIER, petitioner, vs.HON. COMMISSION ON ELECTIONS and RONALD ALLAN KELLEY POE, ALSO KNOWN AS
FERNANDO POE JR., respondents.

On 31 December 2003, respondent Ronald Allan Kelly Poe, also known as Fernando Poe, Jr. (hereinafter "FPJ"), filed his certificate of
candidacy for the position of President of the Republic of the Philippines under the Koalisyon ng Nagkakaisang Pilipino (KNP) Party, in the
forthcoming national elections. In his certificate of candidacy, FPJ, representing himself to be a natural-born citizen of the Philippines, stated
his name to be "Fernando Jr.," or "Ronald Allan" Poe, his date of birth to be 20 August 1939 and his place of birth to be Manila.
On 31 December 2003, respondent Ronald Allan Kelly Poe, also known as Fernando Poe, Jr. (hereinafter "FPJ"), filed his certificate of
candidacy for the position of President of the Republic of the Philippines under the Koalisyon ng Nagkakaisang Pilipino (KNP) Party, in the
forthcoming national elections. In his certificate of candidacy, FPJ, representing himself to be a natural-born citizen of the Philippines, stated
his name to be "Fernando Jr.," or "Ronald Allan" Poe, his date of birth to be 20 August 1939 and his place of birth to be Manila.

Fonacier submitted evidence to support his claim including a birth certificate of FPj, among others. FPJ as respondent presented twenty-two
documentary pieces of evidence, the more significant ones being - a) a certification issued by Estrella M. Domingo of the Archives Division
of the National Archives that there appeared to be no available information regarding the birth of Allan F. Poe in the registry of births for San
Carlos, Pangasinan among others.

On 23 January 2004, the COMELEC dismissed SPA No. 04-003 for lack of merit. Three days later, or on 26 January 2004, Fornier filed his
motion for reconsideration. The motion was denied on 06 February 2004 by the COMELEC en banc. On 10 February 2004, petitioner
assailed the decision of the COMELEC before this Court conformably with Rule 64, in relation to Rule 65, of the Revised Rules of Civil
Procedure.

The other petitions, later consolidated with G. R. No. 161824, would include G. R. No. 161434, entitled "Maria Jeanette C. Tecson, and
Felix B. Desiderio, Jr., vs. The Commission on Elections, Ronald Allan Kelley Poe (a.k.a. Fernando Poe, Jr.), and Victorino X. Fornier,"
and the other, docketed G. R. No. 161634, entitled "Zoilo Antonio G. Velez, vs. Ronald Allan Kelley Poe, a.k.a. Fernando Poe, Jr.," both
challenging the jurisdiction of the COMELEC and asserting that, under Article VII, Section 4, paragraph 7, of the 1987 Constitution, only
the Supreme Court had original and exclusive jurisdiction to resolve the basic issue on the case.

Both petitioner and respondent submitted the same documents as evidence. From the documents submitted, the only conclusions that could
be drawn with some degree of certainty from the documents would be that -
1. The parents of FPJ were Allan F. Poe and Bessie Kelley;
2. FPJ was born to them on 20 August 1939;
3. Allan F. Poe and Bessie Kelley were married to each other on 16 September, 1940;
4. The father of Allan F. Poe was Lorenzo Poe; and
5. At the time of his death on 11 September 1954, Lorenzo Poe was 84 years old.
Petitioners Argument:

Petitioner submits, in any case, that in establishing filiation (relationship or civil status of the child to the father [or mother]) or paternity
(relationship or civil status of the father to the child) of an illegitimate child, FPJ evidently being an illegitimate son according to petitioner,
the mandatory rules under civil law must be used.

In the birth certificate of respondent FPJ, presented by both parties, nowhere in the document was the signature of Allan F. Poe found. There
being no will apparently executed, or at least shown to have been executed, by decedent Allan F. Poe, the only other proof of voluntary
recognition remained to be "some other public document.
Petitioner would have it that even if Allan F. Poe were a Filipino citizen, he could not have transmitted his citizenship to respondent FPJ, the
latter being an illegitimate child. Petitioner contended that as an illegitimate child, FPJ so followed the citizenship of his mother, Bessie
Kelley, an American citizen.

Respondents Argument:
Respondent submitted as proof of other document to support his claim of legitimacy, since it was not indicated in the birth certificate, a
duly notarized declaration made by Ruby Kelley Mangahas, sister of Bessie Kelley Poe as Exhibit 20 before the COMELEC. The
declaration of Ruby Kelly Mangahas among others include the following:
"2. Bessie Kelley Poe was the wife of Fernando Poe, Sr.
"3. Fernando and Bessie Poe had a son by the name of Ronald Allan Poe, more popularly known in the Philippines as `Fernando Poe, Jr., or
`FPJ.
"18. I am executing this Declaration to attest to the fact that my nephew, Ronald Allan Poe is a natural born Filipino, and that he is the
legitimate child of Fernando Poe, Sr.

Issue:

Whether or not FPJ is considered a natural-born Filipino citizen?


Whether or not FPJ is qualified to run for presidency?

Held:

Under the Civil Code of Spain, which was in force in the Philippines from 08 December 1889 up until the day prior to 30 August 1950 when
the Civil Code of the Philippines took effect, acknowledgment was required to establish filiation or paternity. Acknowledgment was either
judicial (compulsory) or voluntary. Judicial or compulsory acknowledgment was possible only if done during the lifetime of the putative
parent; voluntary acknowledgment could only be had in a record of birth, a will, or a public document.32 Complementary to the new code
was Act No. 3753 or the Civil Registry Law expressing in Section 5 thereof, that -
"In case of an illegitimate child, the birth certificate shall be signed and sworn to jointly by the parents of the infant or only by the mother if
the father refuses. In the latter case, it shall not be permissible to state or reveal in the document the name of the father who refuses to
acknowledge the child, or to give therein any information by which such father could be identified."

In the birth certificate of respondent FPJ, presented by both parties, nowhere in the document was the signature of Allan F. Poe found. There
being no will apparently executed, or at least shown to have been executed, by decedent Allan F. Poe, the only other proof of voluntary
recognition remained to be "some other public document.

The 1950 Civil Code categorized the acknowledgment or recognition of illegitimate children into voluntary, legal or compulsory. Voluntary
recognition was required to be expressly made in a record of birth, a will, a statement before a court of record or in any authentic writing.
Legal acknowledgment took place in favor of full blood brothers and sisters of an illegitimate child who was recognized or judicially
declared as natural. Compulsory acknowledgment could be demanded generally in cases when the child had in his favor any evidence to
prove filiation. Unlike an action to claim legitimacy which would last during the lifetime of the child, and might pass exceptionally to the
heirs of the child, an action to claim acknowledgment, however, could only be brought during the lifetime of the presumed parent.

The Family Code has further liberalized the rules; Article 172, Article 173, and Article 175 provide:
"Art. 172. The filiation of legitimate children is established by any of the following:
"(1) The record of birth appearing in the civil register or a final judgment; or
"(2) An admission of legitimate filiation in a public document or a private handwritten instrument and signed by the parent concerned.
"In the absence of the foregoing evidence, the legitimate filiation shall be proved by:
"(1) The open and continuous possession of the status of a legitimate child; or
"(2) Any other means allowed by the Rules of Court and special laws.
"Art. 173. The action to claim legitimacy may be brought by the child during his or her lifetime and shall be transmitted to the heirs should
the child die during minority or in a state of insanity. In these cases, the heirs shall have a period of five years within which to institute the
action.
"The action already commenced by the child shall survive notwithstanding the death of either or both of the parties.
"x x x xxx x x x.
"Art. 175. Illegitimate children may establish their illegitimate filiation in the same way and on the same, evidence as legitimate children.
"The action must be brought within the same period specified in Article 173, except when the action is based on the second paragraph of
Article 172, in which case the action may be brought during the lifetime of the alleged parent."
The provisions of the Family Code are retroactively applied; Article 256 of the code reads:
"Art. 256. This Code shall have retroactive effect insofar as it does not prejudice or impair vested or acquired rights in accordance with the
Civil Code or other laws."
It should be apparent that the growing trend to liberalize the acknowledgment or recognition of illegitimate children is an attempt to break
away from the traditional idea of keeping well apart legitimate and non-legitimate relationships within the family in favor of the greater
interest and welfare of the child. The provisions are intended to merely govern the private and personal affairs of the family. There is little, if
any, to indicate that the legitimate or illegitimate civil status of the individual would also affect his political rights or, in general, his
relationship to the State.

The fact of the matter perhaps the most significant consideration is that the 1935 Constitution, the fundamental law prevailing on the day,
month and year of birth of respondent FPJ, can never be more explicit than it is. Providing neither conditions nor distinctions, the
Constitution states that among the citizens of the Philippines are "those whose fathers are citizens of the Philippines." There utterly is no
cogent justification to prescribe conditions or distinctions where there clearly are none provided.

But while the totality of the evidence may not establish conclusively that respondent FPJ is a natural-born citizen of the Philippines, the
evidence on hand still would preponderate in his favor enough to hold that he cannot be held guilty of having made a material
misrepresentation in his certificate of candidacy in violation of Section 78, in relation to Section 74, of the Omnibus Election Code.
Petitioner has utterly failed to substantiate his case before the Court, notwithstanding the ample opportunity given to the parties to present
their position and evidence, and to prove whether or not there has been material misrepresentation.

Ruling:

WHEREFORE, the Court RESOLVES to DISMISS


1. G. R. No. 161434, entitled "Maria Jeanette C. Tecson and Felix B. Desiderio, Jr., Petitioners, versus Commission on Elections, Ronald
Allan Kelley Poe (a.k.a. "Fernando Poe, Jr.,) and Victorino X. Fornier, Respondents," and G. R. No. 161634, entitled "Zoilo Antonio Velez,
Petitioner, versus Ronald Allan Kelley Poe, a.k.a. Fernando Poe, Jr., Respondent," for want of jurisdiction.
2. G. R. No. 161824, entitled "Victorino X. Fornier, Petitioner, versus Hon. Commission on Elections and Ronald Allan Kelley Poe, also
known as Fernando Poe, Jr.," for failure to show grave abuse of discretion on the part of respondent Commission on Elections in dismissing
the petition in SPA No. 04-003.

No Costs.
SO ORDERED

Shorter version of what was held. This is from a digest I found online.

It is necessary to take on the matter of whether or not respondent FPJ is a natural-born citizen, which, in turn, depended on whether or not
the father of respondent, Allan F. Poe, would have himself been a Filipino citizen and, in the affirmative, whether or not the alleged
illegitimacy of respondent prevents him from taking after the Filipino citizenship of his putative father. Any conclusion on the Filipino
citizenship of Lorenzo Pou could only be drawn from the presumption that having died in 1954 at 84 years old, Lorenzo would have been
born sometime in the year 1870, when the Philippines was under Spanish rule, and that San Carlos, Pangasinan, his place of residence upon
his death in 1954, in the absence of any other evidence, could have well been his place of residence before death, such that Lorenzo Pou
would have benefited from the "en masse Filipinization" that the Philippine Bill had effected in 1902. That citizenship (of Lorenzo Pou), if
acquired, would thereby extend to his son, Allan F. Poe, father of respondent FPJ

6. BAR MATTER
RE: APPLICATION FOR ADMISSION TO THE PHILIPPINE BAR, vs. VICENTE D. CHING, applicant.

FACTS
Vicente D. Ching, the legitimate son of the spouses Tat Ching, a Chinese citizen, and Prescila A. Dulay, a
Filipino, was born in Francia West, Tubao, La Union on 11 April 1964. Since his birth, Ching has resided in
the Philippines.
Ching, after having completed a Bachelor of Laws course at the St. Louis University in Baguio City, filed an
application to take the 1998 Bar Examinations. In a Resolution of this Court, he was allowed to take the Bar
Examinations, subject to the condition that he must submit to the Court proof of his Philippine citizenship.
In compliance, Ching submitted (1) certification showing that he is a public accountant (2) Voters certification
(3) Certification, showing that Ching was elected as a member of the Sangguniang Bayan of Tubao.
On 5 April 1999, the results of the 1998 Bar Examinations were released and Ching was one of the successful
Bar examinees. The oath-taking of the successful Bar examinees was scheduled on 5 May 1999. However,
because of the questionable status of Ching's citizenship, he was not allowed to take his oath.
He was required to submit further proof of his citizenship. In the same resolution, the Office of the Solicitor
General (OSG) was required to file a comment on Ching's petition for admission to the bar and on the
documents evidencing his Philippine citizenship.
The OSG adds that "(w)hat he acquired at best was only an inchoate Philippine citizenship which he could
perfect by election upon reaching the age of majority
In conclusion, the OSG points out that Ching has not formally elected Philippine citizenship and, if ever he
does, it would already be beyond the "reasonable time" allowed by present jurisprudence. However, due to the
peculiar circumstances surrounding Ching's case, the OSG recommends the relaxation of the standing rule on
the construction of the phrase "reasonable period" and the allowance of Ching to elect Philippine citizenship in
accordance with C.A. No. 625 prior to taking his oath as a member of the Philippine Bar.

ISSUE
Whether he has elected Philippine citizenship within a "reasonable time."

RULING

The 1935 Charter provides that the election should be made "upon reaching the age of majority." The age of
majority then commenced upon reaching twenty-one (21) years.
The clause "upon reaching the age of majority" has been construed to mean a reasonable time after reaching
the age of majority which had been interpreted by the Secretary of Justice to be three (3) years
Ching, having been born on 11 April 1964, was already thirty-five (35) years old when he complied with the
requirements of C.A. No. 625 on 15 June 1999, or over fourteen (14) years after he had reached the age of
majority. Based on the interpretation of the phrase "upon reaching the age of majority," Ching's election was
clearly beyond, by any reasonable yardstick, the allowable period within which to exercise the privilege
We hold that Ching failed to validly elect Philippine citizenship. The span of fourteen (14) years that lapsed
from the time he reached the age of majority until he finally expressed his intention to elect Philippine
citizenship is clearly way beyond the contemplation of the requirement of electing "upon reaching the age of
majority." Moreover, Ching has offered no reason why he delayed his election of Philippine citizenship.

7. G.R. No. 142840 May 7, 2001 ANTONIO BENGSON III, petitioner, vs. HOUSE OF REPRESENTATIVES ELECTORAL TRIBUNAL and TEODORO C.
CRUZ, respondents.

KAPUNAN, J., ponente

The citizenship of respondent Teodoro C. Cruz is at issue in this case, in view of the constitutional requirement that "no person shall be a Member of the House of
Representative unless he is a natural-born citizen."

Facts
Congressman Cruz was born on April 27, 1960 in San Clemente, Tarlac, to Filipino parents. He was, therefore, a Filipino citizen, pursuant to Section 1(2), Article IV of the
Constitution. Furthermore, not having done any act to acquire or perfect the Philippine citizenship he obtained from birth, he was a natural-born Filipino citizen, in
accordance with Section 2 of the same Article IV.

Private respondent rendered military service to the United States Marine Corps from November 1958 to October 1993. On June 5, 1990, he was naturalized as an
American citizen, in connection with his US military service. Consequently, under Section 1 (4) of CA No. 63, he lost his Philippine citizenship.

Upon his discharge from the US Marine Corps, private respondent returned to the Philippines and decided to regain his Filipino citizenship.

On March 17, 1994, respondent Cruz reacquired his Philippine citizenship through repatriation under Republic Act No. 2630. He ran for and was elected as the
Representative of the Second District of Pangasinan in the May 11, 1998 elections. He won by a convincing margin of 26,671 votes over petitioner Antonio Bengson III,
who was then running for re-election.

Subsequently, petitioner filed a case for Quo Warranto Ad Cautelam with respondent House of Representatives Electoral Tribunal (HRET) claiming that respondent Cruz
was not qualified to become a member of the House of Representatives since he is not a natural-born citizen as required under Article VI, section 6 of the Constitution.

On March 2, 2000, the HRET rendered its decision dismissing the petition for quo warranto and declaring Cruz the duly elected Representative of the Second District of
Pangasinan in the May 1998 elections. The HRET likewise denied petitioner's motion for reconsideration of the decision in its resolution dated April 27, 2000.

Petitioner thus filed the present petition for certiorari assailing the HRET's decision on the following grounds:

1. The HRET committed serious errors and grave abuse of discretion, amounting to excess of jurisdiction, when it ruled that private respondent is a natural-born
citizen of the Philippines despite the fact that he had ceased being such in view of the loss and renunciation of such citizenship on his part.

2. The HRET committed serious errors and grave abuse of discretion, amounting to excess of jurisdiction, when it considered private respondent as a citizen of the
Philippines despite the fact he did not validly acquire his Philippine citizenship.

3. Assuming that private respondent's acquisition of Philippine citizenship was invalid, the HRET committed serious errors and grave abuse of discretion,
amounting to excess of jurisdiction, when it dismissed the petition despite the fact that such reacquisition could not legally and constitutionally restore his
natural-born status.

Issue
The issue is whether respondent Cruz, a natural-born Filipino who became an American citizen, can still be considered a natural-born Filipino upon his reacquisition of
Philippine citizenship.

Petitioner asserts that respondent Cruz may no longer be considered a natural-born Filipino since he lost his Philippine citizenship when he swore allegiance to
the United States in 1995, and had to reacquire the same by repatriation. He insists that Article citizens are those who are from birth without having to perform
any act to acquire or perfect such citizenship.

Respondent on the other hand contends that he reacquired his status as natural-born citizen when he was repatriated since the phrase "from birth" in Article
IV, Section 2 refers to the innate, inherent and inborn characteristic of being a natural-born citizen.

Ruling
Yes, respondent Cruz, a natural-born Filipino who became an American citizen, can still be considered a natural-born Filipino upon his reacquisition of Philippine
citizenship.
xxx
Repatriation Is Recovery of Original Citizenship

First, repatriation is simply the recovery of original citizenship. Under Section 1 of RA 2630, a person "who ha[s] lost his citizenship" may "reacquire" it by taking an
oath of allegiance to the Republic of the Philippines." Former Senate President Jovito R. Salonga, a noted authority on the subject, explains this method more precisely in
his treatise, Private International Law. He defines repatriation as "the recovery of the original nationality upon fulfilment of certain condition." Webster buttresses this
definition by describing the ordinary or common usage of repatriate, as "to restore or return to one's country of origin, allegiance, or citizenship; x x x." In relation to our
subject matter, repatriation, then, means restoration of citizenship. It is not a grant of a new citizenship, but a recovery of one's former or original citizenship.
xxx
The difference between repatriation and naturalization was also stressed in the case of Angat v. Republic:
xxx

In Angat v. Republic, we held:

xxx. Parenthetically, under these statutes [referring to RA Nos. 965 and 2630], the person desiring to reacquire Philippine citizenship would not even
be required to file a petition in court, and all that he had to do was to take an oath of allegiance to the Republic of the Philippines and to register that
fact with the civil registry in the place of his residence or where he had last resided in the Philippines. (Italics in the original.)

Moreover, repatriation results in the recovery of the original nationality. This means that a naturalized Filipino who lost his citizenship will be restored to his
prior status as a naturalized Filipino citizen. On the other hand, if he was originally a natural-born citizen before he lost his Philippine citizenship, he will be
restored to his former status as a natural-born Filipino.

8. ERNESTO S. MERCADO, petitioner, vs. EDUARDO BARRIOS MANZANO and the COMMISSION ON ELECTIONS, respondents.

[G.R. No. 135083. May 26, 1999.]


Balase, Tamase, Alampay Law Office for petitioner.

Siguion Reyna, Montecillo & Ongsiako for private respondent.

FACTS:

Petitioner Ernesto S. Mercado and private respondent Eduardo B. Manzano were candidates for vice mayor of the City of Makati in the
May 11, 1998 elections. The other one was Gabriel V. Daza III. The results of the election were as follows:

Eduardo B. Manzano103,853

Ernesto S. Mercado100,894

Gabriel V. Daza III54,275 1

The proclamation of private respondent was suspended in view of a pending petition for disqualification filed by a certain Ernesto
Mamaril who alleged that private respondent was not a citizen of the Philippines but of the United States.

In its resolution, dated May 7, 1998, 2 the Second Division of the COMELEC granted the petition of Mamaril and ordered the cancellation
of the certificate of candidacy of private respondent on the ground that he is a dual citizen and, under 40(d) of the Local Government Code,
persons with dual citizenship are disqualified from running for any elective position. The COMELEC's Second Division said:

What is presented before the Commission is a petition for disqualification of Eduardo Barrios Manzano as candidate for
the office of Vice-Mayor of Makati City in the May 11, 1998 elections. The petition is based on the ground that the respondent is an
American citizen based on the record of the Bureau of Immigration and misrepresented himself as a natural-born Filipino citizen.

In his answer to the petition filed on April 27, 1998, the respondent admitted that he is registered as a foreigner with the
Bureau of Immigration under Alien Certificate of Registration No. B-31632 and alleged that he is a Filipino citizen because he was
born in 1955 of a Filipino father and a Filipino mother. He was born in the United States, San Francisco, California, on September
14, 1955, and is considered an American citizen under US Laws. But notwithstanding his registration as an American citizen, he did
not lose his Filipino citizenship.

Judging from the foregoing facts, it would appear that respondent Manzano is both a Filipino and a US citizen. In other
words, he holds dual citizenship.

The question presented is whether under our laws, he is disqualified from the position for which he filed his certificate of
candidacy. Is he eligible for the office he seeks to be elected?

Under Section 40(d) of the Local Government Code, those holding dual citizenship are disqualified from running for any elective
local position.

WHEREFORE, the Commission hereby declares the respondent Eduardo Barrios Manzano DISQUALIFIED as candidate for Vice-
Mayor of Makati City.

On May 8, 1998, private respondent filed a motion for reconsideration. 3 The motion remained pending even until after the election held on May 11,
1998.

Accordingly, pursuant to Omnibus Resolution No. 3044, dated May 10, 1998, of the COMELEC, the board of canvassers tabulated the votes cast for
vice mayor of Makati City but suspended the proclamation of the winner.

On May 19, 1998, petitioner sought to intervene in the case for disqualification. 4 Petitioner's motion was opposed by private respondent.

The motion was not resolved. Instead, on August 31, 1998, the COMELEC en banc rendered its resolution. Voting 4 to 1, with one commissioner
abstaining, the COMELEC en banc reversed the ruling of its Second Division and declared private respondent qualified to run for vice mayor of the City
of Makati in the May 11, 1998 elections. 5 The pertinent portions of the resolution of the COMELEC en banc read:
As aforesaid, respondent Eduardo Barrios Manzano was born in San Francisco, California, U.S.A. He acquired US citizenship by
operation of the United States Constitution and laws under the principle of jus soli.

He was also a natural born Filipino citizen by operation of the 1935 Philippine Constitution, as his father and mother were Filipinos
at the time of his birth. At the age of six (6), his parents brought him to the Philippines using an American passport as travel
document. His parents also registered him as an alien with the Philippine Bureau of Immigration. He was issued an alien certificate
of registration. This, however, did not result in the loss of his Philippine citizenship, as he did not renounce Philippine citizenship
and did not take an oath of allegiance to the United States.

It is an undisputed fact that when respondent attained the age of majority, he registered himself as a voter, and voted in the elections
of 1992, 1995 and 1998, which effectively renounced his citizenship under American law. Under Philippine law, he no longer had
U.S. citizenship.

At the time of the May 11, 1998 elections, the resolution of the Second Division, adopted on May 7, 1998, was not yet final.
Respondent Manzano obtained the highest number of votes among the candidates for vice-mayor of Makati City, garnering one
hundred three thousand eight hundred fifty-three (103,853) votes over his closest rival, Ernesto S. Mercado, who obtained one
hundred thousand eight hundred ninety-four (100,894) votes, or a margin of two thousand nine hundred fifty-nine (2,959) votes.
Gabriel Daza III obtained third place with fifty four thousand two hundred seventy-five (54,275) votes. In applying election laws, it
would be far better to err in favor of the popular choice than be embroiled in complex legal issues involving private international
law which may well be settled before the highest court (Cf. Frivaldo vs. Commission on Elections, 257 SCRA 727).

WHEREFORE, the Commission en banc hereby REVERSES the resolution of the Second Division, adopted on May 7, 1998,
ordering the cancellation of the respondent's certificate of candidacy.

We declare respondent Eduardo Luis Barrios Manzano to be QUALIFIED as a candidate for the position of vice-mayor of Makati
City in the May 11, 1998, elections.

ACCORDINGLY, the Commission directs the Makati City Board of Canvassers, upon proper notice to the parties, to reconvene and
proclaim the respondent Eduardo Luis Barrios Manzano as the winning candidate for vice-mayor of Makati City.

Pursuant to the resolution of the COMELEC en banc, the board of canvassers, on the evening of August 31, 1998, proclaimed private
respondent as vice mayor of the City of Makati. cdasia

This is a petition for certiorari seeking to set aside the aforesaid resolution of the COMELEC en banc and to declare private respondent
disqualified to hold the office of vice mayor of Makati City. Petitioner contends that

[T]he COMELEC en banc ERRED in holding that:

A. Under Philippine law, Manzano was no longer a U.S. citizen when he:

1.He renounced his U.S. citizenship when he attained the age of majority when he was already 37 years old;
and,

2.He renounced his U.S. citizenship when he (merely) registered himself as a voter and voted in the elections
of 1992, 1995 and 1998.

B. Manzano is qualified to run for and or hold the elective office of Vice-Mayor of the City of Makati;

C. At the time of the May 11, 1998 elections, the resolution of the Second Division adopted on 7 May 1998 was not
yet final so that, effectively, petitioner may not be declared the winner even assuming that Manzano is disqualified to run
for and hold the elective office of Vice-Mayor of the City of Makati.

We first consider the threshold procedural issue raised by private respondent Manzano whether petitioner Mercado has personality to
bring this suit considering that he was not an original party in the case for disqualification filed by Ernesto Mamaril nor was petitioner's motion for
leave to intervene granted.

ISSUES:
(1) Whether the petitioner has personality to bring this suit considering that he was not the original party in the disqualification case.

Note: the Supreme Court ruled that under Sec. 6 of R.A. No. 6646, otherwise known as the Electoral Reforms Law of 1987, intervention may
be allowed in proceedings for disqualification even after election if there has yet been no final judgment rendered.

(2) As regards of the issue of citizenship.

NOTE: The Court ruled that by filing a certificate of candidacy when he ran for his present post, private respondent elected Philippine
citizenship and in effect renounced his American citizenship.

RULING:

(1) PETITIONER'S RIGHT TO BRING THIS SUIT

Private respondent cites the following provisions of Rule 8 of the Rules of Procedure of the COMELEC in support of his claim that
petitioner has no right to intervene and, therefore, cannot bring this suit to set aside the ruling denying his motion for intervention:

SECTION 1.When proper and when may be permitted to intervene. Any person allowed to initiate an action or
proceeding may, before or during the trial of an action or proceeding, be permitted by the Commission, in its discretion to intervene
in such action or proceeding, if he has legal interest in the matter in litigation, or in the success of either of the parties, or an interest
against both, or when he is so situated as to be adversely affected by such action or proceeding.

SECTION 3. Discretion of Commission. In allowing or disallowing a motion for intervention, the Commission or the
Division, in the exercise of its discretion, shall consider whether or not the intervention will unduly delay or prejudice the
adjudication of the rights of the original parties and whether or not the intervenor's rights may be fully protected in a separate action
or proceeding.

Private respondent argues that petitioner has neither legal interest in the matter in litigation nor an interest to protect because he is "a defeated
candidate for the vice-mayoralty post of Makati City [who] cannot be proclaimed as the Vice-Mayor of Makati City even if the private respondent
be ultimately disqualified by final and executory judgment."

The flaw in this argument is it assumes that, at the time petitioner sought to intervene in the proceedings before the COMELEC, there had
already been a proclamation of the results of the election for the vice mayoralty contest for Makati City, on the basis of which petitioner came out
only second to private respondent. The fact, however, is that there had been no proclamation at that time. Certainly, petitioner had, and still has, an
interest in ousting private respondent from the race at the time he sought to intervene. The rule in Labo v. COMELEC, 6reiterated in several
cases, 7 only applies to cases in which the election of the respondent is contested, and the question is whether one who placed second to the
disqualified candidate may be declared the winner. In the present case, at the time petitioner filed a "Motion for Leave to File Intervention" on May
20, 1998, there had been no proclamation of the winner, and petitioner's purpose was precisely to have private respondent disqualified "from
running for [an] elective local position" under 40(d) of R.A. No. 7160. If Ernesto Mamaril (who originally instituted the disqualification
proceedings), a registered voter of Makati City, was competent to bring the action, so was petitioner since the latter was a rival candidate for vice
mayor of Makati City.

Nor is petitioner's interest in the matter in litigation any less because he filed a motion for intervention only on May 20, 1998, after private
respondent had been shown to have garnered the highest number of votes among the candidates for vice mayor. That petitioner had a right to
intervene at that stage of the proceedings for the disqualification against private respondent is clear from 6 of R.A. No. 6646, otherwise known as
the Electoral Reforms Law of 1987, which provides:

Any candidate who has been declared by final judgment to be disqualified shall not be voted for, and the votes cast for
him shall not be counted. If for any reason a candidate is not declared by final judgment before an election to be disqualified and he
is voted for and receives the winning number of votes in such election, the Court or Commission shall continue with the trial and
hearing of the action, inquiry, or protest and, upon motion of the complainant or any intervenor, may during the pendency thereof
order the suspension of the proclamation of such candidate whenever the evidence of guilt is strong.

Under this provision, intervention may be allowed in proceedings for disqualification even after election if there has yet been no final
judgment rendered.
The failure of the COMELEC en banc to resolve petitioner's motion for intervention was tantamount to a denial of the motion, justifying
petitioner in filing the instant petition for certiorari. As the COMELEC en banc instead decided the merits of the case, the present petition properly
deals not only with the denial of petitioner's motion for intervention but also with the substantive issues respecting private respondent's alleged
disqualification on the ground of dual citizenship.

This brings us to the next question, namely, whether private respondent Manzano possesses dual citizenship and, if so, whether he is
disqualified from being a candidate for vice mayor of Makati City.

(2) DUAL CITIZENSHIP AS A GROUND FOR DISQUALIFICATION

The disqualification of private respondent Manzano is being sought under 40 of the Local Government Code of 1991 (R.A. No. 7160),
which declares as "disqualified from running for any elective local position: . . . (d) Those with dual citizenship." This provision is incorporated in
the Charter of the City of Makati. 8

Invoking the maxim dura lex sed lex, petitioner, as well as the Solicitor General, who sides with him in this case, contends that through
40(d) of the Local Government Code, Congress has "command[ed] in explicit terms the ineligibility of persons possessing dual allegiance to hold
local elective office."

To begin with, dual citizenship is different from dual allegiance. The former arises when, as a result of the concurrent application of the
different laws of two or more states, a person is simultaneously considered a national by the said states. 9 For instance, such a situation may arise
when a person whose parents are citizens of a state which adheres to the principle of jus sanguinis is born in a state which follows the doctrine
of jus soli. Such a person,ipso facto and without any voluntary act on his part, is concurrently considered a citizen of both states. Considering the
citizenship clause (Art. IV) of our Constitution, it is possible for the following classes of citizens of the Philippines to possess dual citizenship:

(1)Those born of Filipino fathers and/or mothers in foreign countries which follow the principle of jus soli;

(2)Those born in the Philippines of Filipino mothers and alien fathers if by the laws of their fathers' country such children are
citizens of that country;

(3)Those who marry aliens if by the laws of the latter's country the former are considered citizens, unless by their act or omission
they are deemed to have renounced Philippine citizenship.

There may be other situations in which a citizen of the Philippines may, without performing any act, be also a citizen of another state; but
the above cases are clearly possible given the constitutional provisions on citizenship.

Dual allegiance, on the other hand, refers to the situation in which a person simultaneously owes, by some positive act, loyalty to two or
more states. While dual citizenship is involuntary, dual allegiance is the result of an individual's volition.

The record shows that private respondent was born in San Francisco, California on September 4, 1955, of Filipino parents. Since the
Philippines adheres to the principle of jus sanguinis, while the United States follows the doctrine of jus soli, the parties agree that, at birth at least,
he was a national both of the Philippines and of the United States. However, the COMELEC en banc held that, by participating in Philippine
elections in 1992, 1995, and 1998, private respondent "effectively renounced his U.S. citizenship under American law," so that now he is solely a
Philippine national.

Petitioner challenges this ruling. He argues that merely taking part in Philippine elections is not sufficient evidence of renunciation and
that, in any event, as the alleged renunciation was made when private respondent was already 37 years old, it was ineffective as it should have been
made when he reached the age of majority.

In holding that by voting in Philippine elections private respondent renounced his American citizenship, the COMELEC must have in
mind 349 of the Immigration and Nationality Act of the United States, which provided that "A person who is a national of the United States,
whether by birth or naturalization, shall lose his nationality by: . . . (e) Voting in a political election in a foreign state or participating in an election
or plebiscite to determine the sovereignty over foreign territory." To be sure this provision was declared unconstitutional by the U.S. Supreme
Courtin Afroyim v. Rusk 16 as beyond the power given to the U.S. Congress to regulate foreign relations. However, by filing a certificate of
candidacy when he ran for his present post, private respondent elected Philippine citizenship and in effect renounced his American citizenship.
Private respondent's certificate of candidacy, filed on March 27, 1998, contained the following statements made under oath:
6. I AM A FILIPINO CITIZEN (STATE IF "NATURAL-BORN" OR "NATURALIZED") NATURAL-BORN

10. I AM A REGISTERED VOTER OF PRECINCT NO. 747-A, BARANGAY SAN LORENZO, CITY/MUNICIPALITY OF
MAKATI, PROVINCE OF NCR.

11. I AM NOT A PERMANENT RESIDENT OF, OR IMMIGRANT TO, A FOREIGN COUNTRY.

12. I AM ELIGIBLE FOR THE OFFICE I SEEK TO BE ELECTED. I WILL SUPPORT AND DEFEND THE CONSTITUTION
OF THE PHILIPPINES AND WILL MAINTAIN TRUE FAITH AND ALLEGIANCE THERETO; THAT I WILL OBEY THE
LAWS, LEGAL ORDERS AND DECREES PROMULGATED BY THE DULY CONSTITUTED AUTHORITIES OF THE
REPUBLIC OF THE PHILIPPINES; AND THAT I IMPOSE THIS OBLIGATION UPON MYSELF VOLUNTARILY, WITHOUT
MENTAL RESERVATION OR PURPOSE OF EVASION. I HEREBY CERTIFY THAT THE FACTS STATED HEREIN ARE
TRUE AND CORRECT OF MY OWN PERSONAL KNOWLEDGE.

The filing of such certificate of candidacy sufficed to renounce his American citizenship, effectively removing any disqualification he
might have as a dual citizen.

On the other hand, private respondent's oath of allegiance to the Philippines, when considered with the fact that he has spent his youth and
adulthood, received his education, practiced his profession as an artist, and taken part in past elections in this country, leaves no doubt of his election
of Philippine citizenship.

His declarations will be taken upon the faith that he will fulfill his undertaking made under oath. Should he betray that trust, there are
enough sanctions for declaring the loss of his Philippine citizenship through expatriation in appropriate proceedings.

WHEREFORE, the petition for certiorari is DISMISSED for lack of merit.

SO ORDERED.

9. CASE DIGEST: CO VS HRET GR # 92191-92 JULY 30, 1991

FACTS

1. The HRET declared that respondent Jose Ong, Jr. is a natural born Filipino citizen and a resident of Laoang, Northern Samar for voting
purposes. The sole issue before us is whether or not, in making that determination, the HRET acted with grave abuse of discretion.
2. On May 11, 1987, the congressional election for the second district of Northern Samar was held.
3. Among the candidates who vied for the position of representative in the second legislative district of Northern Samar are the petitioners,
Sixto Balinquit and Antonio Co and the private respondent, Jose Ong, Jr.
4. Respondent Ong was proclaimed the duly elected representative of the second district of Northern Samar.
5. The petitioners filed election protests against the private respondent premised on the following grounds:
a. Jose Ong, Jr. is not a natural born citizen of the Philippines; and
b. Jose Ong, Jr. is not a resident of the second district of Northern Samar.
6. The HRET, in its decision dated November 6, 1989, found for the private respondent.
7. A motion for reconsideration was filed by the petitioners on November 12, 1989. This was, however, denied by the HRET, in its
resolution dated February 22, 1989.
8. Hence, these petitions for certiorari.
ISSUES
A. MAIN ISSUES
1. Whether or not the private respondent is a natural born citizen of the Philippines
2. Whether or not the private respondent is a resident of the 2nd district of Northern Samar

B. OTHER ISSUES

1. Whether or not the Supreme Court can take jurisdiction over the case at bar

2. Whether or not grave abuse of discretion was committed by HRET in deciding on the citizenship and residency of the private
respondent

RULINGS

A. CITIZENSHIP

The pertinent portions of the Constitution found in Article IV read:


SECTION 1. The following are citizens of the Philippines:
1. Those who are citizens of the Philippines at the time of the adoption of the Constitution;
2. Those whose fathers or mothers are citizens of the Philippines;
3. Those born before January 17, 1973, of Filipino mothers, who elect Philippine citizenship upon reaching
the age of majority; and
4. Those who are naturalized in accordance with law.
SECTION 2, Natural-born Citizens are those who are citizens of the Philippines from birth without having to perform
any act to acquire or perfect their citizenship. Those who elect Philippine citizenship in accordance with paragraph 3
hereof shall be deemed natural born citizens."

The Court interprets Section 1, Paragraph 3 above as applying not only to those who elect Philippine citizenship after February 2, 1987
but also to those who, having been born of Filipino mothers, elected citizenship before that date.

The provision in Paragraph 3 was intended to correct an unfair position which discriminates against Filipino women. There is no
ambiguity in the deliberations of the Constitutional Commission. The deliberations significantly reveal the intent of the framers. To
make the provision prospective from February 3, 1987 is to give a narrow interpretation resulting in an inequitable situation. It must
also be retroactive.

It should be noted that in construing the law, the Courts are not always to be hedged in by the literal meaning of its language. The spirit
and intendment thereof, must prevail over the letter, especially where adherence to the latter would result in absurdity and injustice.
(Casela v. Court of Appeals, 35 SCRA 279 [1970])

A Constitutional provision should be construed so as to give it effective operation and suppress the mischief at which it is aimed, hence,
it is the spirit of the provision which should prevail over the letter thereof. (Jarrolt v. Mabberly, 103 U.S. 580)

The provision in question was enacted to correct the anomalous situation where one born of a Filipino father and an alien mother was
automatically granted the status of a natural-born citizen while one born of a Filipino mother and an alien father would still have to elect
Philippine citizenship. If one so elected, he was not, under earlier laws, conferred the status of a natural-born.

Under the 1973 Constitution, those born of Filipino fathers and those born of Filipino mothers with an alien father were placed on equal
footing. They were both considered as natural-born citizens.

Hence, the bestowment of the status of "natural-born" cannot be made to depend on the fleeting accident of time or result in two kinds
of citizens made up of essentially the same similarly situated members.
It is for this reason that the amendments were enacted, that is, in order to remedy this accidental anomaly, and, therefore, treat equally
all those born before the 1973 Constitution and who elected Philippine citizenship either before or after the effectivity of that
Constitution.

The Constitutional provision in question is, therefore curative in nature. The enactment was meant to correct the inequitable and absurd
situation which then prevailed, and thus, render those acts valid which would have been nil at the time had it not been for the curative
provisions. (See Development Bank of the Philippines v. Court of Appeals, 96 SCRA 342 [1980])

There is no dispute that the respondent's mother was a natural born Filipina at the time of her marriage. Crucial to this case is the issue
of whether or not the respondent elected or chose to be a Filipino citizen.

Election becomes material because Section 2 of Article IV of the Constitution accords natural born status to children born of
Filipino mothers before January 17, 1973, if they elect citizenship upon reaching the age of majority.

To expect the respondent to have formally or in writing elected citizenship when he came of age is to ask for the unnatural and
unnecessary. The reason is obvious. He was already a citizen. Not only was his mother a natural born citizen but his father had been
naturalized when the respondent was only nine (9) years old. He could not have divined when he came of age that in 1973 and 1987 the
Constitution would be amended to require him to have filed a sworn statement in 1969 electing citizenship inspite of his already having
been a citizen since 1957. In 1969, election through a sworn statement would have been an unusual and unnecessary procedure for one
who had been a citizen since he was nine years old.

We have jurisprudence that defines "election" as both a formal and an informal process.

In the case of In Re: Florencio Mallare (59 SCRA 45 [1974]), the Court held that the exercise of the right of suffrage and the
participation in election exercises constitute a positive act of election of Philippine citizenship. In the exact pronouncement of the
Court, we held:

Esteban's exercise of the right of suffrage when he came of age, constitutes a positive act of election of Philippine
citizenship.

The private respondent did more than merely exercise his right of suffrage. He has established his life here in the Philippines.

For those in the peculiar situation of the respondent who cannot be expected to have elected citizenship as they were already citizens,
we apply the In Re Mallare rule.

An election of Philippine citizenship presupposes that the person electing is an alien. Or his status is doubtful because he is a national of
two countries. There is no doubt in this case about Mr. Ong's being a Filipino when he turned twenty-one (21).

We repeat that any election of Philippine citizenship on the part of the private respondent would not only have been superfluous but it
would also have resulted in an absurdity. How can a Filipino citizen elect Philippine citizenship?

Moreover, the respondent traces his natural born citizenship through his mother, not through the citizenship of his father. The citizenship
of the father is relevant only to determine whether or not the respondent "chose" to be a Filipino when he came of age. At that time and
up to the present, both mother and father were Filipinos. Respondent Ong could not have elected any other citizenship unless he first
formally renounced Philippine citizenship in favor of a foreign nationality. Unlike other persons faced with a problem of election, there
was no foreign nationality of his father which he could possibly have chosen.

There is another reason why we cannot declare the HRET as having committed manifest grave abuse of discretion. The same issue of
natural-born citizenship has already been decided by the Constitutional Convention of 1971 and by the Batasang Pambansa convened by
authority of the Constitution drafted by that Convention. Emil Ong, full blood brother of the respondent, was declared and accepted as a
natural born citizen by both bodies.

The Constitutional Convention was the sole judge of the qualifications of Emil Ong to be a member of that body. The HRET, by
explicit mandate of the Constitution, is the sole judge of the qualifications of Jose Ong, Jr. to be a member of Congress. Both
bodies deliberated at length on the controversies over which they were sole judges. Decisions were arrived at only after a full
presentation of all relevant factors which the parties wished to present. Even assuming that we disagree with their conclusions, we
cannot declare their acts as committed with grave abuse of discretion. We have to keep clear the line between error and grave abuse.

B. RESIDENCE

The petitioners question the residence qualification of respondent Ong.

The petitioners lose sight of the meaning of "residence" under the Constitution. The term "residence" has been understood as
synonymous with domicile not only under the previous Constitutions but also under the 1987 Constitution.

The framers of the Constitution adhered to the earlier definition given to the word "residence" which regarded it as having the same
meaning as domicile.

The term "domicile" denotes a fixed permanent residence to which when absent for business or pleasure, one intends to return. (Ong
Huan Tin v. Republic, 19 SCRA 966 [1967]) The absence of a person from said permanent residence, no matter how long,
notwithstanding, it continues to be the domicile of that person. In other words, domicile is characterized by animus revertendi. (Ujano v.
Republic, 17 SCRA 147 [1966])

The domicile of origin of the private respondent, which was the domicile of his parents, is fixed at Laoang, Samar. Contrary to the
petitioners' imputation, Jose Ong, Jr. never abandoned said domicile; it remained fixed therein even up to the present.

The petitioners' allegation that since the private respondent owns no property in Laoang, Samar, he cannot, therefore, be a resident of
said place is misplaced.

To require the private respondent to own property in order to be eligible to run for Congress would be tantamount to a property
qualification. The Constitution only requires that the candidate meet the age, citizenship, voting and residence requirements. Nowhere is
it required by the Constitution that the candidate should also own property in order to be qualified to run. (see Maquera v. Borra, 122
Phil. 412 [1965])

C. JURISDICTION AND GRAVE ABUSE OF DISCRETION

The Constitution explicitly provides that the House of Representatives Electoral Tribunal (HRET) and the Senate Electoral Tribunal
(SET) shall be the sole judges of all contests relating to the election, returns, and qualifications of their respective members. (See Article
VI, Section 17, Constitution).

The authority conferred upon the Electoral Tribunal is full, clear and complete. The use of the word sole emphasizes the exclusivity of
the jurisdiction of these Tribunals.

The Supreme Court under the 1987 Constitution, has been given an expanded jurisdiction, so to speak, to review the decisions of the
other branches and agencies of the government to determine whether or not they have acted within the bounds of the Constitution. (See
Article VIII, Section 1, Constitution)

Yet, in the exercise thereof, the Court is to merely check whether or not the governmental branch or agency has gone beyond the
Constitutional limits of its jurisdiction, not that it erred or has a different view. In the absence of a showing that the HRET has
committed grave abuse of discretion amounting to lack of jurisdiction, there is no occasion for the Court to exercise its corrective
power; it will not decide a matter which by its nature is for the HRET alone to decide. (See Marcos v. Manglapus, 177 SCRA 668
[1989]) It has no power to look into what it thinks is apparent error.

As constitutional creations invested with necessary power, the Electoral Tribunals, although not powers in the tripartite scheme of the
government, are, in the exercise of their functions independent organs independent of Congress and the Supreme Court. The power
granted to HRET by the Constitution is intended to be as complete and unimpaired as if it had remained originally in the legislature.
(Angara v. Electoral Commission, 63 Phil. 139 [1936])
In passing upon petitions, the Court with its traditional and careful regard for the balance of powers, must permit this exclusive privilege
of the Tribunals to remain where the Sovereign authority has place it. (See Veloso v. Boards of Canvassers of Leyte and Samar, 39 Phil.
886 [1919])

It has been argued that under Article VI, Section 17 of the present Constitution, the situation may exist as it exists today where there is
an unhealthy one-sided political composition of the two Electoral Tribunals. There is nothing in the Constitution, however, that makes
the HRET because of its composition any less independent from the Court or its constitutional functions any less exclusive. The degree
of judicial intervention should not be made to depend on how many legislative members of the HRET belong to this party or that party.
The test remains the same manifest grave abuse of discretion.

In the case at bar, the Court finds no improvident use of power, no denial of due process on the part of the HRET which will necessitate
the exercise of the power of judicial review by the Supreme Court.

WHEREFORE, the petitions are hereby DISMISSED. The questioned decision of the house of Representatives Electoral Tribunal is
AFFIRMED. Respondent Jose Ong, Jr. is declared a natural-born citizen of the Philippines and a resident of Laoang, Northern Samar.

SO ORDERED.

10. CASE : G.R. No. L-21289 October 4, 1971 MOY YA LIM YAO alias EDILBERTO AGUINALDO LIM and LAU YUEN
YEUNG, petitioners-appellants, vs. THE COMMISSIONER OF IMMIGRATION, respondent-appellee.

Facts of the Case:

1. In the instant case, petitioners seek the issuance of a writ of injunction against the Commissioner of Immigration, "restraining the latter
and/or his authorized representative from ordering plaintiff Lau Yuen Yeung to leave the Philippines and causing her arrest and
deportation and the confiscation of her bond, upon her failure to do so."

2. On February 8, 1961, Lau Yuen Yeung, a Chinese residing at Kowloon, Hongkong, applied for a passport visa to enter the Philippines
as a non-immigrant for one month which expires on April 13, 1961.

3. On her arrival , Asher Y, Cheng filed a bond worth P1,000.00 to undertake, among others that said Lau Yuen Yeung would actually
depart from the Philippines on or before the expiration of her authorized period of stay in this country or within the period as in his
discretion the Commissioner of Immigration or his authorized representative might properly allow.

4. After repeated extensions, petitioner Lau Yuen Yeung was allowed to stay in the Philippines up to February 13, 1962.

5. On January 25, 1962, she contracted marriage with Moy Ya Lim Yao alias Edilberto Aguinaldo Lim an alleged Filipino citizen.

6. Because of the contemplated action of respondent to confiscate her bond and order her arrest and immediate deportation, after the
expiration of her authorized stay, she brought this action for injunction with preliminary injunction.

7. The lower court upon deciding the case dated March 19, 1962 invoked:
First, Section 15 of the Revised Naturalization Law provides: = alien woman deemed a citizen of the Philippines b virtue of her
marriage to a Filipino citizen only if she possess all the qualifications and none of the disqualifications specified in the law. However, as
based on testimonies, although not possessing disqualification, doesnt have all the qualifications i.e. lacks requisite length of
residence in the Philippines (Revised Naturalization Law, Sec 2, Case No. 2, Sec. 3, Case No. 3).

Second, Courts found that marriage was effected merely for convenience to defeat or avoid her then impending compulsory departure,
not to say deportation. This cannot be permitted.

Third, as the Solicitor General has well stated: as a temporary alien visitor promising to leave after a period of one month cannot go
back on her representation to stay permently without first departing from the Philippines as she had promised

And fourth, respondent Commissioner of Immigration (Sec. 3, Com. Act No. 613) in the performance of his duties in relation to alien
immigrants, has a wide discretion, a quasi-judicial function in determining cases presented to so that his decision thereon may not be
disturbed unless he acted with abuse of discretion or in excess of his jurisdiction.

8. Appellants alleged that:

Appellants contention/ assignment of errors Supreme Courts contention


The clause who might herself be lawfully naturalized implies thta an That under Section 15 of Commonwealth Act 473, the Revised
alien woman may be deemed a citizen of the Philippines by virtue of Naturalization Law, the marriage of an alien woman to a Filipino
her marriage to a Filipino citizen, only if she possesses all the makes her a Filipina, if she "herself might be lawfully naturalized
qualifications and none of the disqualifications specified in the law The limitation of Section 15 of the Naturalization Law excludes from
the benefits of naturalization by marriage, only those disqualified
from being naturalized under Section 4 of the law
That evidence proving that she is not disqualified may be presented in
the action to recover her bond confiscated by the Commissioner of
Immigration;
That upon proof of such fact, she may be recognized as Filipina

That in referring to the disqualification enumerated in the law, the


Court somehow left the impression that no inquiry need be made
as to qualifications

An alien who does not possess any disqualifications and who married In other words, the applicable statute itself more than implies that
a Filipino citizen is still considered an alien the naturalization of an alien visitor as a Philippine citizen
logically produces the effect of conferring upon him ipso facto all
the rights of citizenship including that of being entitled to
permanently stay in the Philippines outside the orbit of authority
of the Commissioner of Immigration vis-a-vis aliens, if only
because by its very nature and express provisions, the
Immigration Law is a law only for aliens and is inapplicable to
citizens of the Philippines.
in construing the provision of the United States statutes from
which our law has been copied, 28a the American courts have
held that the alien wife does not acquire American citizenship by
choice but by operation of law
That it was a marriage of convenience
Failing to find that the Commissioner of Immigration acted with Does not apply to the case at bar: as it is premised on the assumption
abuse of discretion or in excess of his jurisdiction when he threatened that petitioner is not a Filipino citizen
to send her out and confiscate her bond, arrest and deport her despite Accordingly, it is but safe to assume that were the Solicitor General
being a Filipino citizen and His Honor of the view that said petitioner had become ipso
facto a Filipina by virtue of her marriage to her Filipino husband,
they would have held her as entitled to assume the status of a
permanent resident without having to depart as required of aliens
by Section 9 (g) of the law.

Refusal to permanently enjoin Commissioner from ordering appellant Permanently enjoined from causing the arrest and deportation and the
to leave as a temporary visitor confiscation of the bond of appellant Lau Yuen Yeung, who is hereby
declared to have become a Filipino citizen from and by virtue of her
marriage to her co-appellant Moy Ya Lim Yao alias Edilberto
Aguinaldo Lim
Refusal to grant motion for preliminary injunctions embodied in the The judgment rendered by lower court is reversed
complaint
Further note:

a. The alien woman must file a petition for the cancellation of her alien certificate of registration alleging, among other things, that she is
married to a Filipino, citizen and that she is not disqualified from acquiring her husband's citizenship pursuant to section 4 of
Commonwealth Act No. 473, as amended.

b. Upon the filing of said petition, which should be accompanied or supported by the joint affidavit of the petitioner and her Filipino
husband to the effect that the petitioner does not belong to any of the groups disqualified by the cited section from becoming naturalized
Filipino citizen (please see attached CEB Form 1), the Bureau of Immigration conducts an investigation and thereafter promulgates its
order or decision granting or denying the petition.

c. Once the Commissioner of Immigration cancels the subject's registration as an alien, there will probably be less difficulty in establishing
her Filipino citizenship

ISSUE:

That petitioner Lau Yuen Yeung, having been admitted as a temporary alien visitor on the strength of a deliberate and voluntary
representation that she will enter and stay only for a period of one month and thereby secured a visa, cannot go back on her
representation to stay permanently without first departing from the Philippines as she had promised. (Chung Tiao Bing, et al. vs.
Commissioner of Immigration, G.R. No. L-9966, September 29, 1956; Ong Se Lun vs. Board of Commissioners, G.R. No. L-6017,
Sept. 16, 1954, Sec. 9, last par. Phil. Immigration Law);

That the mere marriage of a Filipino citizen to an alien does not automatically confer on the latter Philippine citizenship. The alien wife
must possess all the qualifications required by law to become a Filipino citizen by naturalization and none of the disqualifications. (Lee
Suan Ay, Alberto Tan and Lee Chiao vs. Galang, etc., G. R. No. L-11855, Dec. 25, 1959)

DECISION

HELD: the judgment of the Court a quo dismissing appellants' petition for injunction is hereby REVERSED and the Commissioner of
Immigration and/or his authorized representative is permanently enjoined from causing the arrest and deportation and the confiscation of the
bond of appellant Lau Yuen Yeung, who is hereby declared to have become a Filipino citizen from and by virtue of her marriage to her co-
appellant Moy Ya Lim Yao alias Edilberto Aguinaldo Lim, a Filipino citizen on January 25, 1962. No costs.
12. LIMKAICHONG vs COMELEC G.R. Nos. 178831-32, July 30, 2009

Facts:

In its April 1, 2009 Decision in G.R. No. 179120, the Supreme Court reversed the Joint Resolution of the COMELEC Second Division dated
May 17, 2007 in SPA Nos. 07-247 and 07-248 disqualifying Limkaichong from running as a congressional candidate in the First District
of Negros Oriental due to lack of citizenship requirement. Biraogo filed the instant motion for reconsideration with prayer for oral argument.

The core issue in the consolidated petitions is the qualification of Limkaichong to run for, be elected to, and assume and discharge, the position
of Representative for the First District of Negros Oriental. The contention of the parties who sought her disqualification is that she is not a
natural-born citizen, hence, she lacks the citizenship requirement in Section 6, Article VI of the 1987 Constitution. In the election that ensued,
she was voted for by the constituents of Negros Orientaland garnered the highest votes. She was eventually proclaimed as the winner and has
since performed her duties and responsibilities as Member of the House of Representatives.

The proponents against Limkaichong's qualification stated that she is not a natural-born citizen because her parents were Chinese citizens at the
time of her birth. They went on to claim that the proceedings for the naturalization of Julio Ong Sy, her father, never attained finality due to
procedural and substantial defects.

Issues:

1. Whether the citizenship of Limkaichong's parents may be questioned in an election case


2. Who has jurisdiction over the disqualification case
3. Whether the ten-day prescriptive period under the 1998 HRET Rules apply to disqualification based on citizenship

Held:

1. No. In assailing the citizenship of the father, the proper proceeding should be in accordance with Section 18 of Commonwealth Act No. 473.
As early as the case of Queto v. Catolico, the Court held that:
It may be true that, as alleged by said respondents, that the proceedings for naturalization were tainted with certain infirmities, fatal or
otherwise, but that is beside the point in this case. The jurisdiction of the court to inquire into and rule upon such infirmities must be properly
invoked in accordance with the procedure laid down by law. Such procedure is the cancellation of the naturalization certificate. In the manner
fixed in Section 18 of Commonwealth Act No. 473, hereinbefore quoted, namely, "upon motion made in the proper proceedings by the Solicitor
General or his representatives, or by the proper provincial fiscal." In other words, the initiative must come from these officers, presumably after
previous investigation in each particular case.

Clearly, under law and jurisprudence, it is the State, through its representatives designated by statute, that may question the illegally or invalidly
procured certificate of naturalization in the appropriate denaturalization proceedings. It is plainly not a matter that may be raised by private
persons in an election case involving the naturalized citizens descendant.

Accordingly, proper proceedings must be strictly followed by the proper officers under the law. Hence, in seeking Limkaichong's disqualification
on account of her citizenship, the rudiments of fair play and due process must be observed, for in doing so, she is not only deprived of the right to
hold office as a Member of the House of Representative but her constituents would also be deprived of a leader in whom they have put their trust
on through their votes.

2. Limkaichong was proclaimed by the Provincial Board of Canvassers, she had taken her oath of office, and she was allowed to officially
assume the office on July 23, 2007. Accordingly, the House of Representatives Electoral Tribunal (HRET), and no longer the COMELEC, should
now assume jurisdiction over the disqualification cases.
x x x The Court has invariably held that once a winning candidate has been proclaimed, taken his oath, and assumed office as a Member of the
House of Representatives, the COMELEC's jurisdiction over election contests relating to his election, returns, and qualifications ends, and the
HRET's own jurisdiction begins.
The fact that the proclamation of the winning candidate, as in this case, was alleged to have been tainted with irregularity does not divest the
HRET of its jurisdiction.

3. No. The 1998 HRET Rules, as amended, provide for the manner of filing either an election protest or a petition for quo warranto against a
Member of the House of Representatives. In our Decision, we ruled that the ten-day prescriptive period under the 1998 HRET Rules does not
apply to disqualification based on citizenship, because qualifications for public office are continuing requirements and must be possessed not
only at the time of appointment or election or assumption of office but during the officer's entire tenure. Once any of the required qualifications is
lost, his title may be seasonably challenged. Accordingly, the 1987 Constitution requires that Members of the House of Representatives must be
natural-born citizens not only at the time of their election but during their entire tenure. Being a continuing requirement, one who assails a
member's citizenship or lack of it may still question the same at any time, the ten-day prescriptive period notwithstanding.

WHEREFORE, the Motion for Reconsideration with Prayer for Oral Argument filed by petitioner Louis C. Biraogo in G.R. No. 179120
is DENIED with FINALITY.

SO ORDERED.

C.A. 473
Sec. 18. Cancellation of Naturalization Certificate Issued. - Upon motion made in the proper proceedings by the Solicitor General or his
representative, or by the proper provincial fiscal, the competent judge may cancel the naturalization certificate issued and its
registration in the Civil Register:

1. If it is shown that said naturalization certificate was obtained fraudulently or illegally;

2. If the person naturalized shall, within five years next following the issuance of said naturalization certificate, return to his native country or to
some foreign country and establish his permanent residence there: Provided, That the fact of the person naturalized remaining more than one year
in his native country or the country of his former nationality, or two years in any other foreign country, shall be considered as prima facie
evidence of his intention of taking up his permanent residence in the same:

3. If the petition was made on an invalid declaration of intention;

4. If it is shown that the minor children of the person naturalized failed to graduate from a public or private high school recognized by the Office
of Private Education [now Bureau of Private Schools] of the Philippines, where Philippine history, government or civics are taught as part of
the school curriculum, through the fault of their parents either by neglecting to support them or by transferring them to another school or schools.
A certified copy of the decree canceling the naturalization certificate shall be forwarded by the Clerk of Court of the Department of Interior [now
Office of the President] and the Bureau of Justice [now Office of the Solicitor General];

5. If it is shown that the naturalized citizen has allowed himself to be used as a dummy in violation of the constitutional or legal provisions
requiring Philippine citizenship as a requisite for the exercise, use or enjoyment of a right, franchise or privilege.
13. TEODULO M. COQUILLA, petitioner, vs. THE HON. COMMISSION ON ELECTIONS and MR. NEIL M. ALVAREZ

Facts:

Petitioner Teodulo M. Coquilla was born of Filipino parents in Oras, Eastern Samar, where he grew up and resided. He was subsequently
naturalized as a U.S. citizen when he joined the US Navy

Petitioner applied for repatriation and took his oath as a citizen of the Philippines on November 10, 2000 after being approved.

On February 27, 2001, he filed his certificate of candidacy stating therein that he had been a resident of Oras, Eastern Samar for "two (2) years."

On March 5, 2001, respondent Neil M. Alvarez, who was the incumbent mayor of Oras and who was running for reelection, sought the
cancellation of petitioners certificate of candidacy on the ground that the latter had made a material misrepresentation in his certificate of
candidacy by stating that he had been a resident of Oras for two years when in truth he had resided therein for only about six months since
November 10, 2000, when he took his oath as a citizen of the Philippines.

The COMELEC was unable to render judgment on the case before the elections on May 14, 200 and May 17, 2001, petitioner was proclaimed
mayor of Oras by the Municipal Board of Canvassers.8 He subsequently took his oath of office.

ISSUE: (On Citizenship)

Whether or not petitioner satisfied the residency requirement for the position of the mayor?

Held:

On the merits, the question is whether petitioner had been a resident of Oras, Eastern Samar at least one (1) year before the elections held on May
14, 2001 as he represented in his certificate of candidacy. We find that he had not.

First, 39(a) of the Local Government Code (R.A No. 7160) provides:

Qualifications. - (a) An elective local official must be a citizen of the Philippines; a registered voter in the barangay, municipality, city, or
province or, in the case of a member of the sangguniang panlalawigan, sangguniang panlungsod, or sangguniang bayan, the district where he
intends to be elected; a resident therein for at least one (1) year immediately preceding the day of the election; and able to read and write Filipino
or any other local language or dialect. (Emphasis added)

The term "residence" is to be understood not in its common acceptation as referring to "dwelling" or "habitation,"21but rather to "domicile" or
legal residence,22 that is, "the place where a party actually or constructively has his permanent home, where he, no matter where he may be
found at any given time, eventually intends to return and remain (animus manendi)."23 A domicile of origin is acquired by every person at birth.
It is usually the place where the childs parents reside and continues until the same is abandoned by acquisition of new domicile (domicile of
choice

In the case at bar, petitioner lost his domicile of origin in Oras by becoming a U.S. citizen after enlisting in the U.S. Navy in 1965. From then on
and until November 10, 2000, when he reacquired Philippine citizenship, petitioner was an alien without any right to reside in the Philippines
save as our immigration laws may have allowed him to stay as a visitor or as a resident alien.

The statement in petitioners certificate of candidacy that he had been a resident of Oras, Eastern Samar for "two years" at the time he filed such
certificate is not true. The question is whether the COMELEC was justified in ordering the cancellation of his certificate of candidacy for this
reason. We hold that it was. Petitioner made a false representation of a material fact in his certificate of candidacy, thus rendering such certificate
liable to cancellation

In the case at bar, what is involved is a false statement concerning a candidates qualification for an office for which he filed the certificate of
candidacy. This is a misrepresentation of a material fact justifying the cancellation of petitioners certificate of candidacy. The cancellation of
petitioners certificate of candidacy in this case is thus fully justified.
WHEREFORE, the petition is DISMISSED and the resolution of the Second Division of the Commission on Elections, dated July 19, 2001, and
the order, dated January 30, 2002 of the Commission on Elections en bancare AFFIRMED.

14. FRIVALDO VS. COMELEC G.R. No. 104654 June 6, 1994

REPUBLIC OF THE PHILIPPINES, petitioner,


vs.
HON. ROSALIO G. DE LA ROSA, PRESIDING JUDGE OF THE REGIONAL TRIAL COURT, BRANCH 28,
MANILA and JUAN G. FRIVALDO, respondents.

Facts:

This is a petition for certiorari under Rule 45 of the Revised Rules of Court in relation to R.A. No. 5440 and Section 25
of the Interim Rules, filed by the Republic of the Philippines: (1) to annul the Decision of the Regional Trial Court,
Branch 28, Manila, which re-admitted private respondent as a Filipino citizen under the Revised Naturalization Law
(C.A. No. 63 as amended by C.A. No. 473); and (2) to nullify the oath of allegiance taken by private respondent on
February 27, 1992.

On September 20, 1991, petitioner filed a petition for naturalization captioned to be re-admitted as citizen of the
Philippines. The respondent Judge set the petition for hearing on March 16, 1992, and directed the publication of the said
order and petition in the Official Gazette and a newspaper of general circulation, for three consecutive weeks, the last
publication of which should be at least six months before the said date of hearing.
On January 14, 1992, private respondent filed a "Motion to Set Hearing Ahead of Schedule, that it shall be done on
January instead of having it on March, where he manifested his intention to run for public office in the May 1992
elections. The motion was granted and the hearing was moved on February 27.Six days later, on February 27, respondent
Judge rendered the assailed Decision and held that Petitioner JUAN G. FRIVALDO, is re-admitted as a citizen of the
Republic of the Philippines by naturalization, thereby vesting upon him, all the rights and privileges of a natural born
Filipino citizen .

After receiving a copy of the Decision on March 18, 1992, the Solicitor General interposed a timely appeal directly with
the Supreme Court.

Issue:
WON the petitioner was duly re-admitted o his citizenship as Filipino.

Held:

No. The supreme court ruled that Private respondent is declared NOT a citizen of the Philippines and therefore
DISQUALIFIED from continuing to serve as GOVERNOR of the Province of Sorsogon. He is ordered to VACATE his
office and to SURRENDER the same to the Vice-Governor of the Province of Sorsogon once this decision becomes final
and executory. No pronouncement as to costs.

The proceedings of the trial court was marred by the following irregularities: (1) the hearing of the petition was set ahead
of the scheduled date of hearing, without a publication of the order advancing the date of hearing, and the petition itself;
(2) the petition was heard within six months from the last publication of the petition; (3) petitioner was allowed to take
his oath of allegiance before the finality of the judgment; and (4) petitioner took his oath of allegiance without observing
the two-year waiting period.
Comments:
G.R. No. 105715is a petition for certiorari, mandamus with injunction for a petition sought to annul the proclamation of
private respondent as Governor-elect of the Province of Sorsogon.

G.R. No. 105735is a petition for mandamus for the cancellation of private respondents certificate of candidacy by the
Comelec.

15. CASE DIGEST TABASA VS CA GR 125793 August 29, 2006

FACTS

1. Joevanie Arellano Tabasa was a natural-born citizen of the Philippines. In 1968, when petitioner was seven years old, his
father, Rodolfo Tabasa, became a naturalized citizen of the United States. By derivative naturalization (citizenship derived
from that of another as from a person who holds citizenship by virtue of naturalization), petitioner also acquired American
citizenship.

2. Petitioner arrived in the Philippines on August 3, 1995, and was admitted as a "balikbayan" for one year. Petitioner was then
arrested and detained and brought to the BID Detention Center in Manila.

3. Petitioner was investigated and accused of violating Section 8, Chapter 3, Title 1, Book 3 of the 1987 Administrative Code,
in a charge sheet which alleged:

a. That on 3 August 1995, respondent (petitioner herein [Tabasa]) arrived in the Philippines and was admitted
as a balikbayan;

b. That in a letter dated 16 April 1996, Honorable Kevin Herbert, Consul General of [the] U.S. Embassy,
informed the Bureau that respondent's Passport No. 053854189 issued on June 10, 1994 in San Francisco,
California, U.S.A., had been revoked by the U.S. Department of State;

c. Hence, respondent [petitioner Tabasa] is now an undocumented and undesirable alien and may be
summarily deported pursuant to Law and Intelligence Instructions No. 53 issued by then Commissioner
Miriam Defensor Santiago to effect his deportation (Exhibit 3).

4. The BID ordered petitioner's deportation to his country of origin, the United States, on May 29, 1996.

5. Petitioner filed before the CA a Petition for Habeas Corpus with Preliminary Injunction and/or Temporary
Restraining Order on May 29, 1996. Tabasa alleged that he was not afforded due process; that no warrant of arrest
for deportation may be issued by immigration authorities before a final order of deportation is made; that no notice
of the cancellation of his passport was made by the U.S. Embassy; that he is entitled to admission or to a change of
his immigration status as a non-quota immigrant because he is married to a Filipino citizen as provided in Section
13, paragraph (a) of the Philippine Immigration Act of 1940; and that he was a natural-born citizen of the
Philippines prior to his derivative naturalization when he was seven years old due to the naturalization of his father,
Rodolfo Tabasa, in 1968.

6. At the time Tabasa filed said petition, he was already 35 years old.

7. However, on June 13, 1996, petitioner filed a Supplemental Petition alleging that he had acquired Filipino
citizenship by repatriation in accordance with Republic Act No. 8171 (RA 8171), and that because he is now a
Filipino citizen, he cannot be deported or detained by the respondent Bureau.
8. The CA, in its August 7, 1996 Decision, denied Tabasa's petition on the ground that he had not legally and
successfully acquired by repatriation his Filipino citizenship as provided in RA 8171.

a. The court said that although he became an American citizen by derivative naturalization when his father
was naturalized in 1968, there is no evidence to show that he lost his Philippine citizenship "on account of
political or economic necessity," as explicitly provided in Section 1, RA 8171 the law governing the
repatriation of natural-born Filipinos who have lost their citizenship. The affidavit does not state that
political or economic necessity was the compelling reason for petitioner's parents to give up their Filipino
citizenship in 1968.

b. Moreover, the court a quo found that petitioner Tabasa did not dispute the truth of the April 16, 1996 letter
of the United States Consul General Kevin F. Herbert or the various warrants issued for his arrest by the
United States court.

c. The court a quo noted that after petitioner was ordered deported by the BID on May 29, 1996, he
successively executed an Affidavit of Repatriation on June 6, 1996 and took an oath of allegiance to the
Republic of the Philippines on June 13, 1996 more than ten months after his arrival in the country on
August 3, 1995.

d. The appellate court considered petitioner's "repatriation" as a last ditch effort to avoid deportation and
prosecution in the United States. The appellate court concluded that his only reason to want to reacquire
Filipino citizenship is to avoid criminal prosecution in the United States of America.

e. The court a quo, therefore, ruled against Tabasa, whose petition is now before us.

ISSUES
1. Whether or not the petitioner has validly reacquired Philippine citizenship under RA 8171. If there is no valid repatriation,
then he can be summarily deported for his being an undocumented alien.
RULING
1. The Court finds no merit in this petition.

RA 8171, "An Act Providing for the Repatriation of Filipino Women Who Have Lost Their Philippine Citizenship by
Marriage to Aliens and of Natural-Born Filipinos," was enacted on October 23, 1995. It provides for the repatriation of
only two (2) classes of persons, viz:

Filipino women who have lost their Philippine citizenship by marriage to aliens and natural-born
Filipinos who have lost their Philippine citizenship, including their minor children, on account of
political or economic necessity, may reacquire Philippine citizenship through repatriation in the manner
provided in Section 4 of Commonwealth Act No. 63, as amended: Provided, That the applicant is not a:

1. Person opposed to organized government or affiliated with any association or group of persons
who uphold and teach doctrines opposing organized government;

2. Person defending or teaching the necessity or propriety of violence, personal assault, or


association for the predominance of their ideas;

3. Person convicted of crimes involving moral turpitude; or

4. Person suffering from mental alienation or incurable contagious diseases.


Does petitioner Tabasa qualify as a natural-born Filipino who had lost his Philippine citizenship by reason of
political or economic necessity under RA 8171?

He does not.

Petitioner theorizes that he could be repatriated under RA 8171 because he is a child of a natural-born Filipino, and that he
lost his Philippine citizenship by derivative naturalization when he was still a minor.
Petitioner overlooks the fact that the privilege of repatriation under RA 8171 is available only to natural-born Filipinos who
lost their citizenship on account of political or economic necessity, and to the minor children of said natural-born Filipinos.
This means that if a parent who had renounced his Philippine citizenship due to political or economic reasons later decides
to repatriate under RA 8171, his repatriation will also benefit his minor children according to the law. This includes a
situation where a former Filipino subsequently had children while he was a naturalized citizen of a foreign country. The
repatriation of the former Filipino will allow him to recover his natural-born citizenship and automatically vest Philippine
citizenship on his children of jus sanguinis or blood relationship: the children acquire the citizenship of their parent(s) who
are natural-born Filipinos. To claim the benefit of RA 8171, however, the children must be of minor age at the time the
petition for repatriation is filed by the parent. This is so because a child does not have the legal capacity for all acts of civil
life much less the capacity to undertake a political act like the election of citizenship. On their own, the minor children
cannot apply for repatriation or naturalization separately from their parents.
In the case at bar, there is no dispute that petitioner was a Filipino at birth. In 1968, while he was still a minor, his father
was naturalized as an American citizen; and by derivative naturalization, petitioner acquired U.S. citizenship. Petitioner
now wants us to believe that he is entitled to automatic repatriation as a child of natural-born Filipinos who left the country
due to political or economic necessity. This is absurd. Petitioner was no longer a minor at the time of his "repatriation" on
June 13, 1996. The privilege under RA 8171 belongs to children who are of minor age at the time of the filing of the
petition for repatriation.
Neither can petitioner be a natural-born Filipino who left the country due to political or economic necessity. Clearly, he lost
his Philippine citizenship by operation of law and not due to political or economic exigencies. It was his father who could
have been motivated by economic or political reasons in deciding to apply for naturalization. The decision was his parent's
and not his. The privilege of repatriation under RA 8171 is extended directly to the natural-born Filipinos who could prove
that they acquired citizenship of a foreign country due to political and economic reasons, and extended indirectly to the
minor children at the time of repatriation.
In sum, petitioner is not qualified to avail himself of repatriation under RA 8171. However, he can possibly reacquire
Philippine citizenship by availing of the Citizenship Retention and Re-acquisition Act of 2003 (Republic Act No. 9225) by
simply taking an oath of allegiance to the Republic of the Philippines.
FILING FOR PETITION UNDER RA 8171
Even if we concede that petitioner Tabasa can avail of the benefit of RA 8171, still he failed to follow the procedure for
reacquisition of Philippine citizenship. He has to file his petition for repatriation with the Special Committee on
Naturalization (SCN)
What petitioner simply did was that he took his oath of allegiance to the Republic of the Philippines; then, executed an
affidavit of repatriation, which he registered, together with the certificate of live birth, with the Office of the Local Civil
Registrar of Manila. The said office subsequently issued him a certificate of such registration. At that time, the SCN was
already in place and operational by virtue of the June 8, 1995 Memorandum issued by President Fidel V. Ramos.
REQUIREMENTS FOR REPATRIATION UNDER RA 8171
Even if petitioner now of legal age can still apply for repatriation under RA 8171, he nevertheless failed to prove that
his parents relinquished their Philippine citizenship on account of political or economic necessity as provided for in the law.

Repatriation is not a matter of right, but it is a privilege granted by the State. This is mandated by the 1987
Constitution under Section 3, Article IV, which provides that citizenship may be lost or reacquired in the manner provided
by law. The State has the power to prescribe by law the qualifications, procedure, and requirements for repatriation. It has
the power to determine if an applicant for repatriation meets the requirements of the law for it is an inherent power of the
State to choose who will be its citizens, and who can reacquire citizenship once it is lost. If the applicant, like petitioner
Tabasa, fails to comply with said requirements, the State is justified in rejecting the petition for repatriation.

Petitioner claims that because of his repatriation, he has reacquired his Philippine citizenship; therefore, he is not an
undocumented alien subject to deportation.
This theory is incorrect.
As previously explained, petitioner is not entitled to repatriation under RA 8171 for he has not shown that his case falls
within the coverage of the law.
Petitioner Tabasa, whose passport was cancelled after his admission into the country, became an undocumented alien who
can be summarily deported. His subsequent "repatriation" cannot bar such deportation especially considering that he has no
legal and valid reacquisition of Philippine citizenship.
WHEREFORE, this petition for review is DISMISSED, and the August 7, 1996 Decision of the Court of Appeals is AFFIRMED.
No costs to the petitioner.
SO ORDERED.

16. Case Digest on ANGAT V. REPUBLIC

14 Sept.1999

Facts: Petitioner was a natural born citizen who lost his citizenship by naturalization in the US. On March 11, 1996, he
filed a petition with the RTC to regain his status as a citizen of the Philippines. The court thereafter repatriated Petitioner.

Issue: Whether the RTC has jurisdiction over repatriation cases

Held: No. A petition for repatriation should be filed with the Special Committee on Naturalization and not with the
RTC which has no jurisdiction there over. The courts order was thereby null and void. The Special Committee on
Naturalization was reactivated on June 8, 1995, hence, when Petioner filed his petition on March 11, 1996, the
Committee constituted pursuant to LOI No. 270 under PD No. 725 (a Decree providing for repatriation of Filipino
women who had lost their Philippine citizenship by marriage to aliens and of natural born Filipinos) was in place.

SYNOPSIS

Petitioner Gerardo Angat was a natural born citizen of the Philippines. He lost his citizenship by naturalization in the
United States of America. In 1991, he returned to the Philippines. On March 11, 1996, he filed before the Regional Trial
Court (RTC) of Marikina City a petition to regain his status as a citizen of the Philippines. On May 10, 1996, the Office
of the Solicitor General (OSG) was notified of the initial hearing scheduled on January 27, 1997. However, on
September 20, 1996, upon motion of the petitioner, he was allowed to take the Oath of Allegiance to the Republic of the
Philippines which was scheduled on October 3, 1996. On October 4, 1996, the trial judge issued an Order declaring the
petitioner as repatriated and a citizen of the Republic of the Philippines pursuant to Republic Act No. 8171. On March
19, 1997, the OSG filed a Motion asserting that the petition itself should have been dismissed by the court a quo for lack
of jurisdiction because the proper forum was the Special Committee on Naturalization consistent with Administrative
Order No. 285. On September 22, 1997, the trial court set aside its Orders dated September 20, 1996 and October 04,
1996 and dismissed the petition on the ground of lack of jurisdiction without prejudice to its refilling before the Special
Committee on Naturalization.

Hence, this appeal by certiorari.

The Court ruled that when petitioner filed his petition on March 11, 1996, the Special Committee on Naturalization
constituted pursuant to LOI No. 270 under P.D. No. 725 was in place. Administrative Order 285, promulgated on August
22, 1996 relative to R.A. No. 8171, in effect, was merely then a confirmatory issuance. The Office of the Solicitor
General was right in maintaining that Angat's petition should have been filed with the Committee, aforesaid, and not with
the RTC which had no jurisdiction thereover. The court's order of October 4, 1996 was thereby null and void, and it did
not acquire finality nor could be a source of right on the part of petitioner.

The petition for review was DENIED.

17. Digest on Altajeros vs Comelec

Facts:

Ciceron P. Altarejos, a candidate for mayor in the Municipality of San Jacinto, Masbate in the May 10, 2004 national and
local elections was petitioned by the respondents to be disqualified or cancel his certificate of candidacy on the ground
that he is not a Filipino citizen and made a false representation in his certificate of candidacy that he was not a permanent
resident of or immigrant to a foreign country.

Issues:

1) Is the registration of petitioners repatriation with the proper civil registry and with the Bureau of Immigration a
prerequisite in effecting repatriation; and 2) Whether or not the COMELEC en banc committed grave abuse of discretion
amounting to excess or lack of jurisdiction in the affirming the Resolution of the COMELEC, First Division in
disqualifying Altajeros in his candidacy.

Rulings:

1) Yes. The registration of the Certificate of Repatriation in the proper civil registry and the Bureau of Immigration is a
prerequisite in effecting the repatriation of a citizen. In the case at bar, petitioner completed all the requirements of
repatriation only after he filed his certificate of candidacy for a mayoralty position but before the elections. Petitioners
repatriation retroacted to the date he filed his application and was, therefore, qualified to run for a mayoralty position in
the government in the May 10, 2004 elections.
2) No. Petitioner submitted necessary documents proving compliance with the requirements of repatriation only during
his motion for reconsideration, which could no longer be considered as evidence by the COMELEC en banc in The
Comelec Rules of Procedures.

Petition DENIED.

18. LOIDA NICOLAS-LEWIS, GREGORIO B. MACABENTA, ALEJANDRO A. ESCLAMADO, ARMANDO B. HEREDIA,


REUBEN S. SEGURITAN, ERIC LACHICA FURBEYRE, TERESITA A. CRUZ, JOSEFINA OPENA DISTERHOFT,
MERCEDES V. OPENA, CORNELIO R. NATIVIDAD, EVELYN D. NATIVIDAD,
Petitioners, V

COMMISSION ON ELECTIONS, Respondent.


G.R. No. 162759 August 4, 2006

FOR QUICK REFERENCE DURING ORALS:

RA 9225: RA 9189:
Citizenship Retention and Re-Acquisition Act of Overseas Absentee Voting Act of 2003 (OAVL)
2003
- Recognition of Philippine Citizenship = right - COMELECaccordingly ordered to allow
to suffrage them to vote and register as absentee voters
- Acquirement of civil and political rights and
liabilities

Article V of Constitution RA 9189 (OAVL) RA 9225:


Sec. 1: Residency requirement as factor A response to Article V of Constitution Sec. 3: Retention of Phil. citizenship
for the right to vote
Requires voter must be resident in the Sec. 4. Coverage. All citizens of natural-born citizens of the Philippines
Phil. For at least 1 yr and in the place the Philippines abroad, who are not otherwise who have lost their Philippine citizenship
where he proposes to vote for at least 6 disqualified by law, at least eighteen (18) by reason of their naturalization as
mos. Immediately preceding an election years of age on the day of elections, may citizens of a foreign country are hereby
deemed to have re-acquired Philippine
citizenship upon taking the following oath
of allegiance to the Republic:
Sec 2: Congress shall provide.... a system Sec. 5. Disqualifications. SEC. 4. Derivative Citizenship.
of absentee voting
Therefore, no residency requirement (d) An immigrant or a permanent resident The unmarried child, whether legitimate,
who is recognized as such in the host country, illegitimate or adopted, below eighteen
unless he/she executes, upon registration, an (18) years of age, of those who re-
affidavit prepared for the purpose by the acquire Philippine citizenship upon
Commission declaring that he/she shall effectivity of this Act shall be deemed
resume actual physical permanent residence citizens of the Philippines.
in the Philippines not later than three (3) SEC. 5. Civil and Political Rights and
years from approval of his/her registration Liabilities.
under this Act. Such affidavit shall also state
that he/she has not applied for citizenship in Those who retain or re-acquire
another country. Failure to return shall be the Philippine citizenship under this Act
cause for the removal of the name of the shall enjoy full civil and political rights
immigrant or permanent resident from the and be subject to all attendant liabilities
National Registry of Absentee Voters and and responsibilities under existing laws
his/her permanent disqualification to vote in of the Philippines and the following
absentia. conditions

HELD BY COURT AS
CONSTITUTIONAL:
As the execution of the affidavit explicitly
shows he/she DID NOT ABANDON
domicile of origin.

Senator Arroyo (Anti - voting) Senator Angara (PRO - voting)


Section 1, Article V, of the Constitution . Interpretation of RESIDENCE is synonymous with
DOMICILE with intent to go home
the Constitution says, who shall have resided in
the Philippines. They are permanent immigrants. They have Sec. 2 of Article V as strategically placed immediately after
changed residence so they are barred under the Constitution. Sec. 1 with indicates it as an exception
It is legally and constitutionally impossible to give a franchise
to vote to overseas Filipinos who do not physically live in the
country, which is quite ridiculous
because that is exactly the whole point of this exercise to
enfranchise them and empower them to vote.
FACTS:
In this petition for certiorari and mandamus, petitioners, referring to themselves as "duals" or dual citizens, pray that they and others who
retained or reacquired Philippine citizenship under Republic Act (R.A.) No. 9225, the Citizenship Retention and Re-Acquisition Act of
2003, be allowed to avail themselves of the mechanism provided under the Overseas Absentee Voting Act of 2003 [1] (R.A. 9189) and that
the Commission on Elections (COMELEC) accordingly be ordered to allow them to vote and register as absentee voters under the aegis of
R.A. 9189.

Thus, Long before the May 2004 national and local elections, petitioners sought registration and certification
as "overseas absentee voter" only to be advised by the Philippine Embassy in the United States that, per a COMELEC letter to the
Department of Foreign Affairs dated September 23, 2003[2],
they have yet no right to vote in such elections owing to their lack of the one-year residence requirement prescribed by the
Constitution. The same letter, however, urged the different Philippine posts abroad not to discontinue their campaign for voters
registration, as the residence restriction adverted to would contextually affect merely certain individuals who would likely be eligible to
vote in future elections.

According to the COMELEC, OAVL was not enacted for them (petitioners) for they merely acquired their citizenship making them a
regular voter who has to meet the requirements of residency under Sec. 1 of Article V of the Constitution. Thus petitioners filed this
petition for certiorari and mandamus on April 1 of 2004. In the same sense, the Office of the Solcitor Gerenal (OSG) said that voters may
exercise their right to vote right after only the issue had become moot and academic.

ISSUE:
WON Petitioners and others who might have meanwhile retained and/or reacquired Philippine citizenship pursuant to RA 9225
may vote as absentee voter under RA 9189.

RULING:
COURT DISAGRESS WITH THE COMELEC.

1) THERE IS NO PROVISION IN THE DUAL CITIZENSHIP LAW - R.A. 9225 - REQUIRING "DUALS" TO
ACTUALLY ESTABLISH RESIDENCE and physically stay in the Philippines first before they can exercise their right to
vote. On the contrary, R.A. 9225, in implicit acknowledgment that duals are most likely non-residents, grants under its Section
5(1) the same right of suffrage as that granted an absentee voter under R.A. 9189. It cannot be overemphasized that R.A. 9189
aims, in essence, to enfranchise as much as possible all overseas Filipinos who, save for the residency requirements exacted of an
ordinary voter under ordinary conditions, are qualified to vote.
2) BY THE DOCTRINE OF NECESSARY IMPLICATION IN STATUTORY CONSTRUCTION, ,
the strategic location of Section 2 indicates that the Constitutional Commission provided for an exception to the actual
residency requirement of Section 1 with respect to qualified Filipinos abroad. The same Commission has in effect declared that
qualified Filipinos who are not in the Philippines may be allowed to vote even though they do not satisfy the residency
requirement in Section 1, Article V of the Constitution.

Petition is GRANTED. Accordingly, the Court rules and so holds that those who retain or re-acquire Philippine citizenship
under Republic Act No. 9225, the Citizenship Retention and Re-Acquisition Act of 2003, may exercise the right to vote under the system
of absentee voting in Republic Act No. 9189, theOverseas Absentee Voting Act of 2003.

19. CALILUNG VS. DATUMANONG, GR. 160689, MAY 11, 2007

FACTS:

Petitioner:

Petitioner filed the instant petition against respondent, then Secretary of Justice Simeon Datumanong, the official tasked to implement
laws governing citizenship.

Petitioner prays that a writ of prohibition be issued to stop respondent from implementing Republic Act No. 9225, entitled "An Act
Making the Citizenship of Philippine Citizens Who Acquire Foreign Citizenship Permanent, Amending for the Purpose Commonwealth
Act No. 63, As Amended, and for Other Purposes." Petitioner avers that Rep. Act No. 9225 is unconstitutional as it violates Section 5,
Article IV of the 1987 Constitution that states, "Dual allegiance of citizens is inimical to the national interest and shall be dealt with by
law."

Petitioner contends that Rep. Act No. 9225 cheapens Philippine citizenship. He avers that Sections 2 and 3 of Rep. Act No. 9225,
together, allow dual allegiance and not dual citizenship. Petitioner maintains that Section 2 allows all Filipinos, either natural-born or
naturalized, who become foreign citizens, to retain their Philippine citizenship without losing their foreign citizenship. Section 3 permits
dual allegiance because said law allows natural-born citizens of the Philippines to regain their Philippine citizenship by simply taking an
oath of allegiance without forfeiting their foreign allegiance. The Constitution, however, is categorical that dual allegiance is inimical to
the national interest.

Petitioner likewise advances the proposition that although Congress has not yet passed any law on the matter of dual allegiance, such
absence of a law should not be justification why this Court could not rule on the issue. He further contends that while it is true that there
is no enabling law yet on dual allegiance, the Supreme Court, through Mercado v. Manzano,already had drawn up the guidelines on how
to distinguish dual allegiance from dual citizenship.

Office of the Solicitor General(OSG):

The Office of the Solicitor General (OSG) claims that Section 2 merely declares as a state policy that "Philippine citizens who become
citizens of another country shall be deemed not to have lost their Philippine citizenship."

The OSG further claims that the oath in Section 3 does not allow dual allegiance since the oath taken by the former Filipino citizen is an
effective renunciation and repudiation of his foreign citizenship. The fact that the applicant taking the oath recognizes and accepts the
supreme authority of the Philippines is an unmistakable and categorical affirmation of his undivided loyalty to the Republic.

For its part, the OSG counters that pursuant to Section 5, Article IV of the 1987 Constitution, dual allegiance shall be dealt with by law.
Thus, until a law on dual allegiance is enacted by Congress, the Supreme Court is without any jurisdiction to entertain issues regarding
dual allegiance.

ISSUES:

Is Rep. Act No. 9225 unconstitutional?

Does the Court have jurisdiction to pass upon the issue of dual allegiance?

RULING:

Petition dismissed.

To begin with, Section 5, Article IV of the Constitution is a declaration of a policy and it is not a self-executing provision. The legislature still
has to enact the law on dual allegiance. In Sections 2 and 3 of Rep. Act No. 9225, the framers were not concerned with dual citizenship per se,
but with the status of naturalized citizens who maintain their allegiance to their countries of origin even after their naturalization.Congress was
given a mandate to draft a law that would set specific parameters of what really constitutes dual allegiance.Until this is done, it would be
premature for the judicial department, including this Court, to rule on issues pertaining to dual allegiance.

Neither can we subscribe to the proposition of petitioner that a law is not needed since the case of Mercado had already set the guidelines for
determining dual allegiance. Petitioner misreads Mercado. That case did not set the parameters of what constitutes dual allegiance but merely
made a distinction between dual allegiance and dual citizenship.

Moreover, in Estrada v. Sandiganbayan,we said that the courts must assume that the legislature is ever conscious of the borders and edges of its
plenary powers, and passed laws with full knowledge of the facts and for the purpose of promoting what is right and advancing the welfare of the
majority. Hence, in determining whether the acts of the legislature are in tune with the fundamental law, we must proceed with judicial restraint
and act with caution and forbearance.The doctrine of separation of powers demands no less. We cannot arrogate the duty of setting the
parameters of what constitutes dual allegiance when the Constitution itself has clearly delegated the duty of determining what acts constitute
dual allegiance for study and legislation by Congress.

NOTES:

Rep. Act No. 9225, signed into law by President Gloria M. Arroyo on August 29, 2003, reads:

SECTION 1. Short Title.-This Act shall be known as the "Citizenship Retention and Reacquisition Act of 2003."
SEC. 2. Declaration of Policy.-It is hereby declared the policy of the State that all Philippine citizens who become citizens of another
country shall be deemed not to have lost their Philippine citizenship under the conditions of this Act.

SEC. 3. Retention of Philippine Citizenship.-Any provision of law to the contrary notwithstanding, natural-born citizens of the
Philippines who have lost their Philippine citizenship by reason of their naturalization as citizens of a foreign country are hereby
deemed to have reacquired Philippine citizenship upon taking the following oath of allegiance to the Republic:

"I ___________________________, solemnly swear (or affirm) that I will support and defend the Constitution of the Republic of the
Philippines and obey the laws and legal orders promulgated by the duly constituted authorities of the Philippines; and I hereby declare
that I recognize and accept the supreme authority of the Philippines and will maintain true faith and allegiance thereto; and that I impose
this obligation upon myself voluntarily without mental reservation or purpose of evasion."

Natural-born citizens of the Philippines who, after the effectivity of this Act, become citizens of a foreign country shall retain their
Philippine citizenship upon taking the aforesaid oath.

SEC. 4. Derivative Citizenship. - The unmarried child, whether legitimate, illegitimate or adopted, below eighteen (18) years of age, of
those who reacquire Philippine citizenship upon effectivity of this Act shall be deemed citizens of the Philippines.

SEC. 5. Civil and Political Rights and Liabilities. - Those who retain or reacquire Philippine citizenship under this Act shall enjoy full
civil and political rights and be subject to all attendant liabilities and responsibilities under existing laws of the Philippines and the
following conditions:

(1) Those intending to exercise their right of suffrage must meet the requirements under Section 1, Article V of the Constitution,
Republic Act No. 9189, otherwise known as "The Overseas Absentee Voting Act of 2003" and other existing laws;

(2) Those seeking elective public office in the Philippines shall meet the qualifications for holding such public office as required by the
Constitution and existing laws and, at the time of the filing of the certificate of candidacy, make a personal and sworn renunciation of
any and all foreign citizenship before any public officer authorized to administer an oath;

(3) Those appointed to any public office shall subscribe and swear to an oath of allegiance to the Republic of the Philippines and its duly
constituted authorities prior to their assumption of office: Provided, That they renounce their oath of allegiance to the country where
they took that oath;

(4) Those intending to practice their profession in the Philippines shall apply with the proper authority for a license or permit to engage
in such practice; and

(5) That right to vote or be elected or appointed to any public office in the Philippines cannot be exercised by, or extended to, those who:

(a) are candidates for or are occupying any public office in the country of which they are naturalized citizens; and/or

(b) are in the active service as commissioned or noncommissioned officers in the armed forces of the country which they are naturalized
citizens.

SEC. 6. Separability Clause. - If any section or provision of this Act is held unconstitutional or invalid, any other section or provision
not affected thereby shall remain valid and effective.

SEC. 7. Repealing Clause. - All laws, decrees, orders, rules and regulations inconsistent with the provisions of this Act are hereby
repealed or modified accordingly.

SEC. 8. Effectivity Clause. - This Act shall take effect after fifteen (15) days following its publication in the Official Gazette or two (2)
newspapers of general circulation.

You might also like